Текст
                    В.И. Берник, И. К. Жук, О.В. Мельников
СБОРНИК
олимпиаднЫх задач
по математике


В. И. Берник, И. К. Жук, О. В. Мельников СБОРНИК олимпиадных задач по математике Минск «Народная асвета» 1980
ББК 22.1 я 72 Б 51 УДК 51(076.1) Берник В. И. и др. 51 Сборник олимпиадных задач по математике /В. И. Берник, И. К. Жук, О. В. Мельников.—Мн.: Нар. асвета, 1980.— 144 с, ил. В пособие включены задачи различной степени трудности для подготовки и проведения школьных, районных и областных олимпиад по математике. Все задачи снабжены подробными решениями. Сборник адресуется учащимся старших классов. Он может быть использован учителями математики для проведения внеклассной рабоч ты и факультативных занятий. ББК 22.1 я 72 20202—180 51(076) БМ303(05)-80109~79 4306020400 ,-,'tfvp- © Издательство «Народная асвета», 1980,
ОТ АВТОРОВ Возросший в последние годы интерес школьников к точным наукам, и в частности к математике, привел к увеличению числа участников математических олимпиад. Им, а также всем учащимся старших классов, увлекающимся математикой, и адресовано настоящее пособие. Значительную часть сборника составляют задачи, предлагавшиеся в 1975—1978 гг. на областных математических олимпиадах. Кроме того, представлены задачи, которые в течение ряда лет использовались на занятиях школы юных математиков при Институте математики АН БССР, а также Республиканской летней физико-математической школы в пионерском лагере «Зубренок». Все задачи сборника разделены на группы, объединенные либо темой, либо идеей решения. В каждой группе последние задачи более сложные, чем первые. Ко всем задачам даны решения. Большинство задач олимпиадного характера. Однако ряд из них (74, 78, 83, 102, 108, 110, 111, 114, 116, 118, 119, 121, 122, 124) носит обучающий характер. Они призваны ознакомить читателя с некоторыми стандартными фактами и рассуждениями. Задачи 50, 95, 116, 141, 153 являются вспомогательными и используются при решении других, более трудных, задач. Задачи 27 и 28 — наибо-
лее сложные в сборнике. Решение каждой из них — это как бы небольшое математическое исследование. В пособии, как правило, использованы понятия и обозначения, соответствующие школьной программе. В необходимых случаях пояснения даются либо в решении задач, либо в сносках. Все замечания и пожелания по данной книге просим направлять по адресу: 220600 Минск, Парковая магистраль, 11. Издательство «Народная асвета», редакции физики и математики.
ЗАДАЧИ 1. Запишем рациональные положительные числа в виде последовательности: 1 ; 1 ' 2 'J 1 ' 2 ' 3 ; 1 ' 2 ' 3 ' 4 ; ' ' ' ' Найти номер места, на котором стоит . 2. Найти наибольшее нечетное натуральное число, которое нельзя представить в виде суммы трех нераьных составных натуральных чисел. 3. Доказать тождество: 1 + ...+ ... + . 2 3 2л п + \ 2п 4. Натуральный ряд чисел разбит на группы следующим образом: (1), (2, 3), (4, 5, 6), (7, 8, 9, 10), (11, 12, 13, 14, 15), .... Найти, чему равно Sx + S3 + S6 + ... + + S2n-u если Sk — сумма чисел k-й группы. 5. Найти сумму I3 + 53 + 93 + ... + (4п + I)3. 6. Пусть п — натуральное число. Положим *о = — ; xk = —— (хо + х±+ ... + Xk-i), k = п п — k = 1, 2, ... , п— 1. Найти сумму х0 + х± + ... + хл-ь
7. Пусть последовательность Flf F2, ... задана следующим образом: Fx = 1, F2 = — 1, Fn = — Fn-\ — 2/г„_2 при п^З. Доказать, что при п ^2 число г^1 — 7/^ является точным квадратом. 8. Как расставить скобки в выражении 22# (п двоек), чтобы полученное число было максимальным из всех возможных? 9. Доказать, что все шесть выражений агЬ2с3) a2bsclt аф\С<ь —ахЪъс2, —а2Ьхсг, —a3b2cx не могут быть одновременно положительными. 10. 8п— 4 точек расположены в виде креста (рис. 1 для п = 4). Сколькими способами можно выбрать из этих точек четыре, являющиеся вершинами квадрата? 11. Пусть Pi < Р2 < • • • < Рп — различные простые числа. р р Рассмотрим выражение: Расставляя различными спо- • - собами скобки, можем получать разные выражения. Например, при п = 4 можно рас- рис i ставить скобки следующими способами: рх : (р2: (р3: р4)); (Pi: Ра) • (Рз: Р4); Pi: ({р2: ft): P4); (Pi: (P2 • Рз)): ft.
Легко заметить, что первое и последнее из написанных выражений представляют одно и то же число ^^ . Спрашивается, сколько различных чисел можно получить из предложенного выражения, расставляя скобки, при произвольном я^>2? 12. Рассмотрим все наборы с= (аъ ..., ak) из целых чисел, такие, что О^а^/г при i = 1, 2, . .. , k. Обозначим через т (с) минимальное из чисел аь ... , ак. Доказать, что сумма т(с) по всем рассматриваемым выше наборам с равна lk + 2* + ... + *А 13. Доказать, что среди д-значных чисел, в записи которых участвуют лишь цифры 1 и 2, не существует более чисел, каждые два из которых различаются п +1 не менее чем в трех разрядах. 14. В круге проведено п хорд, которые пересекаются внутри круга в т точках, причем точка пересечения хорд считается k раз, если через нее проходит k + 1 хорда. На сколько частей эти хорды делят круг? 15.. Доказать, что при фиксированных натуральных я> by с уравнение ап+Ьп = сп имеет не более одного решения во множестве натуральных чисел п. 16. Доказать, что для любого натурального п уравнение Хх + х2 + ... + хп == хг-х2- ...•*/, имеет по крайней мере одно решение во множестве натуральных чисел. 17. Доказать, что уравнение Ух + Vy = "/"1977 не имеет решений во множестве натуральных чисел. 18. Решить в целых числах уравнение: Зх2 + 5у2 = 345.
19. Найти все решения уравнения "о2" Т2" ~ Т2 во множестве натуральных чисел. 20. Доказать, что при л>1 и различных натуральных alt ... , ап невозможно равенство -L + -L+ -J--L-1 21. Пусть р—простое число. Доказать, что 2^ + Зр нельзя представить в виде хт> где х и т—натуральные числа и т> 1. 22. Найти все решения уравнения 2т—Зп = 1 во множестве натуральных чисел. 23. Найти все решения уравнения 2х + Зу = z2 во множестве целых чисел. 24. Найти все решения уравнения У2 + У = & + х? + х2 + х во множестве целых чисел. 25. Найти все решения уравнения 4х + 4* + 42 = а2 во множестве целых чисел. 26. Решить уравнение X (X2 — У2) Х + 1 во множестве натуральных чисел. 8
27. Найти все решения уравнений во множестве целых чисел: а) х2 (х2 + у2) = ут+\ т > 0, т — целое число; б) х2 (х2 + у) = ут+\ т > 0, т — целое число. 28. Доказать, что система не имеет решений во множестве натуральных чисел. 29. Решить уравнение: Л;3_[*] = 3. 30. Решить уравнение [х2\ = [х]2 во множестве положительных чисел, 31. Решить уравнение: 32. Вычислить сумму [)/Т] + []/2j + \V~Z] + ... + + iVn2 - 1]. 33. Пусть а — действительное число, с — на тральное. Доказать, что 34. Пусть п—целое положительное число. Доказать, что -i-J + ... 35. Для целых а, 6 и m найти сумму 1 (ах Л- у \— ет—1 (ах Л- Ь\ у \—— I, если а и т взаимно просты.
36. Доказать, что [2а + 2р]>[а + р] + :<х] + [р] 37. Решить неравенство [#]{*}<* — 1. 38. Дано, что a1a2 = b1b2 и alt а2, blt b2—положительные числа. Доказать, что 39. По кругу расположены п положительных чисел а>ъ а2> • • • > ап. Обозначим через bt (i = 1, 2, ... , п) сумму т последовательных чисел, начиная с а., расположенных по часовой стрелке. Доказать, что при т < п тп axa2 ... ап ^ bxb2 ... Ьп , причем равенство имеет место лишь в том случае, когда ах = а2 = ... = ап. 40. Пусть х и а—положительные действительные числа, лг>а. Доказать, что для любого натурального п (х—а)п{х + па)<(х2 + па2) 2 . 41. Даны натуральные числа п и N. Найти минимальное число W, представимое в виде W = B1 + B2+ ... + Вп, где Blf В2У ... , Вп —натуральные числа и ВгВ2 ... В ~> 42. Пусть 0<а. < -^, где i = 1, 2, ... , /г. Доказать неравенства: а) sin ах + sin а2 +... + sin ап </г sin Г*1 2+">+V| ; б) sinavsin а2- .... апал <sin 10
43. Доказать, что для любого натурального п lsin п\ , |sin(/i+l)| , , |sin (3/г—1)| 1 44. Пусть m и п—натуральные числа. Доказать, что по крайней мере одно из чисел у^т, у^п не превосходит 45. Доказать, что " -уГп > У п + 1 при п ^ 2. 46. Доказать, что (а ЗаЬ, ) + sin а / \ cos а если а—острый угол, а^О, ^ 47. Пусть jclf х2» - - - t ^„ — такие действительные числа, что х\ + х\ + ... + х\ = ^i + x2+ -..+^)\ Дока. 1 * п п зать, что хх = a:2 = ... = хп . 48. а, 6 и с—действительные числа. Доказать, что по крайней мере одно из чисел (а—б)2, (а—с)2, (Ь—с)2 не превосходит ———-3-— . 49. Пусть аг ^ а2 ^ ... ^ ап — действительные числа. (Л > 1), s = ] + (fl2 — Лз)2 + (Оа —а4)2 + ... + (а2 —ап )2 — aj2. Доказать, что а2 + ...+ ап) — Найти условия, при которых имеет место равенство. 50. Доказать, что для любого натурального К '11
51. Доказать, что для любых натуральных М и N 52. Доказать, что для любых натуральных М и N 53. Пусть Ах2 + Вх + С = (агх + Ъг) (а2* + Ь2). Положим Н = max (|Л|, |Б|, |С|), hx = max fla^, l^l), A2 = = max (\ajit \bj[). Доказать, что 54. Пусть Р (х) = а0 + ахх + ... + апхп — многочлен степени п с действительными коэффициентами иа>3 — действительное число. Доказать, что по крайней мере одно из чисел |1 — Р(0)|, \а—Р(1)\, \а2 — Р(2)|, ... , 1^+*- 1)| не меньше 1. 55. Доказать, что число ((3!)!)! имеет в десятичной записи более тысячи цифр. Определить, на какое число нулей оканчивается это число. 56. Доказать, что число 241977 + 141977 делится на 19. 57. Пусть D— дискриминант квадратного трехчлена ах2 + bx + с с целыми коэффициентами. Доказать, что D не может быть равным ни 1978, ни 1979. 58. Пусть D — некоторое натуральное число, являющееся дискриминантом квадратного трехчлена ах2 + Ьх-\- |сс целыми коэффициентами. Найти минимальное из 12
всех возможных D > О, не являющееся квадратом целого числа. 59. Найти все такие натуральные числа п, что \\Гп\ делит п. 60. Найти все натуральные числа м, делящиеся на все числа, большие или равные ]Лг и не превосходящие п. 61. Доказать, что существует лишь конечное число i натуральных я, делящихся на целые т, ^^ 62. Пусть ах=1, а2=1 и ап+х = ап + аЛ_ь /г > 2 (последовательность Фибоначчи). Доказать, что при лю- бых натуральных &, я дробь —:Lr-r несократима. 63. Пусть множество Р, состоящее из натуральных чисел, обладает следующим свойством: если т, п £ Р, то и т-\- п£Р. Доказать, что для некоторого натурального числа d все числа из Р делятся на d и существует такое /С, что kd$.P для любого kK 64. Доказать, что для любого натурального числа существует кратное ему число, в десятичной записи которого участвуют только цифры 0 и 1. 65. Для произвольного натурального числа т найти три последние цифры числа т100. 66. Доказать, что для произвольного натурального п существует арифметическая прогрессия, состоящая из п составных чисел, все члены которой попарно взаимно просты. 67. Последовательности (alt а2, ... , ап) и (bl9 Ь2, ..., Ьп) из п целых чисел назовем пропорциональными, если найдется такое целое число /, что ах = tblt а^ = tb2, ... , ап = tbn . Доказать, что для любого п > 3 существует бесконечно много попарно не пропорциональных последо- 13
вателыюстей (аи а2, ... , ап) из п целых чисел, для которых справедливо равенство а\ + а\-\- ... + а2п_х = а2п. 68. Назовем пифагоровой тройкой тройку натуральных чисел (х, у, z), такую, что х < у < z и х2 + у2 = г2. Доказать, что для любого натурального п число г^1 встречается ровно в п различных пифагоровых тройках. 69. Доказать, что сумма всех делителей натурального числа п > 2 меньше п ]/1г. 70. Для натуральных х и п обозначим через Nn (x) число таких натуральных d, что d делит л: и x^d2^ < п2. Найти сумму Nn (1) + #л (2) + ... + Nn (я2). 71. Пусть п—натуральное число, большее 1. Обозначим через М множество всех пар натуральных чисел (/?, q), таких, что 1<р<^^^> Р + 9>я, Р и q взаимно просты. Например, при п = 5 М = {(1, 5), (2, 5), (3, 5), (4, 5), (3, 4)}. Доказать, что сумма чисел по всем РЯ (/?, q)£M равна — . 72. Пусть т и п—натуральные числа, /л> 1. Доказать, что УИг можно представить в виде УТп = 1 + ^i -V^m-i—1, где Л^ь ;V2, ... , yVm_! - натуральные числа. 73. Пусть п — натуральное число. Обозначим через f (п) сумму простых делителей nt взятых с их кратно- стями, увеличенную на 1, т. е. если п = рах1 ра22 ... р%п— разложение п на простые множители, то /(/!)= 1 + + aiPi + • • • + атРт' Доказать, что если п > 6, то в последовательности л, /(я), f(/(^)),... всегда встретится число 8. Следовательно, наша последовательность, начиная с какого-то места, принимает вид: 8, 7, 8, 7 14
74. Пусть d — наибольший общий делитель натуральных чисел а и Ь. Доказать, что существуют такие целые тип, что am -f- bn = d. 75. Доказать, что если а и Ь—различные простые числа, то существует бесконечно много таких натуральных чисел п, что числа а + п, 6 + п являются натуральными взаимно простыми. 76. Пусть a, by с—натуральные числа, причем а делит Ь\ а и с взаимно просты; b и с не являются взаимно простыми. Доказать, что существует такое натуральное число d, при котором ad + с и Ь взаимно просты. 77. Доказать, что если alf а2, ... , ап — произвольные целые числа, а т — натуральное число, большее пу то существует такое целое г, что ни одно из чисел #1 + г, а2 + г» • • • , cLn + r не делится на т. 78. Доказать, что для любого натурального числа К существует такое натуральное число Л^>/(, которое нельзя представить в виде суммы не более п — 1 слагаемых, каждое из которых является я-й степенью натурального числа. 79. Доказать, что для любого К существуют такие целые л:>/(, у>К, х2Фу39 для которых выполняется неравенство \х2 —у3\ < х. 80. Найти все простые числа видап ^п ' —1. 81. Доказать, что если число 2п + 1 простое, то п = = 2т 82. Доказать, что существует бесконечно много таких натуральных п, что п2фх2 + Р, где р—простое число, а х —целое. 83. Пусть К—некоторое натуральное число. Доказать, что в последовательности натуральных чисел существует бесконечно много промежутков, содержащих К последо- 15
вательных составных чисел, т. е. промежутков, свободных от простых чисел. 84. Доказать, что для простых р > 5 равенство (р — — 1)1 -}- 1 = рт невозможно ни при каком натуральном т. 85. Найти все простые числа /?, такие, что сумма всех натуральных делителей числа р4 является квадратом целого числа. 86. В арифметической прогрессии первые 15 членов являются простыми числами. Доказать, что разность этой прогрессии больше 30 000. 87. Доказать, что для любого простого числа р найдутся такие целые числа х, у, что х2 + у2 + 1 делится на р. 88. Пусть п — нечетное натуральное число, большее 3. Обозначим через k минимальное натуральное число, такое, что kn + 1—точный квадрат, а через /—минимальное натуральное число, такое, что In—точный квадрат. Доказать, что число п простое тогда и только тогда, когда одновременно k > — и / > — . 4 4 89. Пусть рх < р2 < р3 < ... — последовательность всех простых чисел. Доказать, что между числами рг + + Р2 + •.. + Рп и рг + р2 + ... + Рп+\ всегда найдется точный квадрат. 90. Пусть S—множество из п элементов M(czSy M(=£0y ;= 1, 2, ... , п+ 1. Доказать, что найдутся два различных набора чисел таких, что Mti U Mit[}... U Mif = Ми U Ми[)... U 16
91. Пусть av а2, ... , ап — последовательность из п натуральных чисел. Доказать, что всегда можно выбрать несколько стоящих подряд чисел, сумма которых делится на п. 92. Пусть av а2, ... , ап+i — натуральные числа, причем 1 ^ ах < а2 < ... < а,г+1 < 2п. Доказать, что всегда можно выбрать два таких числа а. и ау, где 1 ^ i < / ^ <л + 1, что ау. делится на а.. 93. Пусть alf а2, ... , ап — натуральные числа, причем 1 ^ аг < а2 ^ ... <! ап и ах + я2 + • • • + ап = 2л. а) Доказать, что при четном п и апфп + I из чисел 01, 02> • • • > я* всегда можно выбрать несколько чисел, сумма которых равна п. б) Доказать, что при нечетном п и при апф2 из чисел аь а2, ... , ап всегда можно выбрать несколько чисел, сумма которых равна п. 94. Пусть А — множество точек на окружности, переходящее в свою часть при повороте на 1 радиан (по часовой стрелке). Доказать, что любая дуга на окружности содержит точки из А. 95. Точку с координатами (хг—х2, у± — у2) назовем разностью двух точек (xly yj, (x2, у2). Доказать, что если площадь фигуры больше 1, то в ней всегда можно найти две точки, разностью которых является целая точка, т. е. точка с координатами (т, я), где т и п—целые числа. 96. Доказать, что любую фигуру, площадь которой больше п, можно так расположить на плоскости, что она закроет п + 1 целых точек. 97. Доказать, что около прямой у = — х Н на рас- 3 5 стоянии, не превосходящем V3a, нет точек М (т, п) с целыми тип. 17
98. Доказать, что для любого а существуют такие целые р и <7>0, что \aq—р\ ^— . з 99. а) Доказать, что для любого целого п^О существует круг на плоскости, содержащий внутри себя ровно п точек с целыми координатами. б) Доказать, что существует такая система /Со, Кх,..., Кпу ... концентрических кругов на плоскости, что для любого п круг Кп содержит внутри себя ровно п точек с целыми координатами. 100. Доказать, что ни при каком целом п>1 число п 2 3 п не является целым. 101. Пусть Доказать, что существует такое п (2^п^ 1000), что 1 nf ^ 251 102. Доказать, что если многочлен с целыми коэффициентами имеет вид хп + ап_х хп"л + ... + ахх + а0, то любой его рациональный корень является целым числом. 103. Пусть Р (х)—многочлен 4-й степени с целыми коэффициентами. Известно, что для любого целого х Р (х) делится на 7. Доказать, что все коэффициенты Р (х) делятся на 7. 18
Привести пример многочлена большей степени, удовлетворяющего условию задачи, у которого не все коэффициенты делятся на 7. 104. Обозначим через Мт (N) число многочленов вида Р (х) = хт+ пт-i хт~1 + ... + ахх + а0, где am_v ат_2, ... , а0 — целые числа, у которых все корни вещественные и по модулю не превосходят N. Доказать, что число Мт (N) конечно. 105. При х = 0, 1, .. . , п — 1 многочлен с целыми коэффициентами принимает значения, отличные от нуля и по модулю меньшие п. Доказать, что многочлен не имеет целых корней. 106. Доказать, что не существует многочлена с целыми положительными коэффициентами, принимающего только простые значения при всех целых значениях аргумента. 107. Пусть рг, р2, ... , рп — фиксированные простые числа. Доказать, что не существует многочлена с целыми положительными коэффициентами, принимающего лишь значения вида ра{* ра2* ... р°пп при всех целых значениях аргумента, где av а2, ... , ап — неотрицательные целые числа. 108. Известно, что а — корень многочлена с целыми коэффициентами. Доказать, что при любом натуральном ГП/ т у а—тоже корень многочлена с целыми коэффициентами. 109. Доказать, что ]/"2 + У^З — корень многочлена о целыми коэффициентами. ПО. Пусть а и р—вещественные корни квадратных трехчленов с целыми коэффициентами. Доказать, что аР— корень многочлена с целыми коэффициентами не выше 4-й степени. 19
111. Пусть а и (3—вещественные корни квадратных трехчленов с целыми коэффициентами. Доказать, что а+ + Р — корень многочлена с целыми коэффициентами не выше 4-й степени. 112. Доказать, что многочлен вида / (л:) = хп — ап-\ хп~~х — ... — а±х — а0, где а0 > 0, ... , ап-\ > 0, а0 + ах + ... + ап-\ > 0, имеет единственный положительный корень. 113. Пусть /(*)= \+х + х* + ... + *m~\ g{x) = = хпх + ха* + .. . + хап , где 0 < ах < а2 < ... < ап - целые числа. Обозначим через пк (k = 0, 1, ... , т — 1) число таких i, что at при делении на т дает в остатке k. Доказать, что g (x) делится на / (х) тогда и только тогда, когда п0 = nv = ... = /im_i. 114. Доказать, что если av a2, ... , ап — различные целые числа, то многочлен (х —аг) (х —а2) ...(х —aj — 1 нельзя представить в виде произведения двух многочленов с целыми коэффициентами. 115. Доказать, что sin Г — число иррациональное. 116. Доказать иррациональность числа 0,1234 ... 101112 ... . 117. Доказать иррациональность числа 0, Р (1) Р (2)... Р (п) ... , где Р (х) —многочлен с целыми коэффициентами, принимающий при х > 0 только положительные значения. Например, при Р (х) = х2 + 1 получим 0,251017.... 118. Число е — основание натуральных логарифмов определяется, как lim (1 -| ) . Доказать, что е — ир- п-*«\ nj 20
рациональное число, используя его другое представление 119. Доказать, что чигло е не может быть корнем никакого квадратного трехчлена с целыми коэффициентами, считая известным, что 120. Доказать, что на плоскости не существует равнобедренного треугольника с углом при вершине 45°, вершины которого находятся в целых точках. 121. Доказать, что при рациональном а внутри интервала ]0, Ц всегда можно найти отрезок, в котором нет членов последовательности ап = {an}, п = 1, 2, ... . 122. Доказать, что при иррациональном а в любом интервале из ]0, 1[ содержится член последовательности ап = {a/г}, /г = 1, 2, .... 123. Доказать, что в любом интервале из ]0, 1[ содержатся члены последовательности ап = {lg п}у п = 1, 2,.. . . 124. Доказать, что при любом натуральном q^2 существует такое действительное число а, что в любом интервале ]а, Ь[ си ]0, 1[ содержится хотя бы один член последовательности ап = {а^}, /г = 1, 2, .... 125. Доказать, что в любом интервале из ]0, 1[ содержатся члены последовательности ап = {}/Гп}у п= 1, 2,.... 126. Дана такая последовательность чисел xv хъ ..., хп> ... , что для любого п 0 < хп < 1 и хп+\ (1 —хп) > > —. Доказать, что lim хп = — . 4 п->°о 2 21
127. Пусть + + ••• + Доказать, что существует lim Snf и вычислить этот предел. 128. Найти lim {(2+1/2)"}. rt-*oo 129. Даны последовательности av а29 ... и bv b2f ..., такие, что ал>0, 6„>0, lim a£ = a>0, lim ^ = b >0. Пусть р, 9 — такие действительные числа, что {р; ^}cz 0 1. Найти lim (pan + qbn)n. П- 130. Пусть функция /(#) монотонно возрастает и удовлетворяет условию «— /(о Доказать, что для любого с>0 lim J^- = 1. <-« /(О 131. В окружность радиуса 1 вписывается квадрат, в квадрат вписывается окружность, в эту окружность — 8-угольник, в него—снова окружность, в окружность — 16-угольник и т. д.; каждый раз в п-ю окружность вписывается 2*+1-угольник, а в него — (п + 1)-я окружность. Пусть Rn — радиус п-й окружности. Чему равен lim Rn? 132. Дано k отличных от нуля чисел alf a2, ... , ak и известно, что при любом нечетном п апх + а% + ... + ank = = 0. Доказать, что k = 2q и числа at, a2> ... 9 а* можно разбить на ^ пар так, что сумма чисел каждой пары равна нулю. 22
133. В некоторые из k ящиков поместили по k ящиков меньшего размера, затем в некоторые из них—вновь по k ящиков и т. д. После этого было подсчитано число ящиков, в которых содержится хотя бы один ящик; их оказалось т. Сколько всего ящиков было использовано? 134. На доске записаны натуральные числа 1, 2, ..., N. Разрешается стереть любые два числа и записать вместо одного из них их разность. Каким должно быть число N> чтобы, повторяя эту операцию, можно было получить на доске одни нули? 135. Для некоторого множества Л, состоящего из натуральных чисел, через А + А обозначим множестио чисел вида пг + п2> где пх и п2 € А. Доказать, что для любого подмножества В множества натуральных чисел найдется такое подмножество А множества натуральных чисел, что либо А + А а В, либо (А + А) П В = 0. 136. Существует ли такое множество М действительных чисел, что для произвольного числа г > 0 и любого элемента а £ М существует такой единственный элемент Ь£ М, что \а—Ь\ =г? 137. Дана таблица 101 х 101 клеток, заполненная целыми числами от 1 до 101, причем в каждом столбце все числа различны. Таблица симметрична относительно диагонали. Доказать, что на этой диагонали все числа различны. 138. Найти внутри выпуклого четырехугольника такую точку, чтобы отрезки, соединяющие ее с серединами сторон, делили площадь четырехугольника на четыре равные части. 139. Пусть ABCD — произвольный четырехугольник. Известно, что окружности, вписанные в треугольники ABC и ACD, касаются. Доказать, что касаются окружности, вписанные в треугольники ABD и BCD. 23
140. Дан ыпуклый четырехугольник ABCD площадью 1. На сторонах АВ и CD отмечены точки А', В' и С, U так, что JM1 = \АВ\ ^ |СС| _ fl №В'\ __ [DPI = |CD| ' |Л5| |CD| = 6, где а + 6< 1 (рис.2). Определить площадь четы- Рис- 2 рехугольника A'B'C'D1. 141. (Теорема Птоломея.) Доказать, что если четырехугольник ABCD вписан в окружность, то \АВ\ • \CD\ + \ВС\ • |i4D| = \АС\. \BD\. 142. Пусть а, 6, с, d — голожительные числа и а> > Ь > с > d. Как надо расположить на плоскости пять точек О, Л, S, С, D, чтобы \ОА\ = а, |ОВ| = 6, |ОС| = с, |OD| = d, a четырехугольник с вершинами в точках Л, В, С, D имел максимальную площадь? 143. Можно ли в произвольный прямоугольник площадью поместить несколько непересекающихся кру- 1978 гов так, чтобы сумма их радиусов была равна 1978? 144. На плоскости расположено бесконечное множество прямоугольников, стороны которых параллельны осям координат, одна из вершин совпадает с началом координат, а длины сторон — целые числа. Доказать, что из этих прямоугольников можно выбрать бесконечную последовательность, в которой каждый многоугольник содержится в следующем за ним. /ч 145. ABCD — параллелограмм (ABC > 90°), О —точка пересечения диагоналей, В'—основание перпендикуляра, 24
опущенного из точки D на диагональ АС, С— основание перпендикуляра, опущенного из D на сторону АВ, А' — основание перпендикуляра, опущенного из D на ВС. Доказать, что точка О лежит на окружности, описанной вокруг АА'В'С. 146. Треугольник ABC вписан в окружность радиуса R^c центром в точке О. Положим &а равным расстоянию от О до (ВС), если точки О и А расположены в одной пол у плоскости относительно прямой (ВС), и этому расстоянию со знаком минус, если О и А расположены в разных полуплоскостях относительно (ВС). Аналогично определяются &в и dc. (Так, напри- Рис- 3 мер, на рисунке 3 dA = \OK\, dB=— \OL\9 dc = \ОМ\.) Пусть г—-радиус вписанной в Д ABC окружности. Доказать, что dA + dB + dc = R + г. 147. В окружность вписан треугольник. Доказать, что основания перпендикуляров, опущенных из произвольной точки, взятой на окружности, на его стороны, лежат на одной прямой. 148. На сторонах Д ABC внешним образом построены квадраты BCDE, ACFG и БАНК. Пусть точки Р и Q таковы, что F С DP и EBKQ — параллелограммы. Доказать, что APAQ — равнобедренный прямоугольный. 149. Дан такой Д АВСУ что А<В, С =90°, О —центр описанной, / — центр вписанной окружности. Найти отношение сторон треугольника, если известно, что Д ВЮ — прямоугольный. 25
150. Дан Л ABC, в котором \ВС\ = a, \AC\ = bt || = с. Доказать, что если прямая, соединяющая точку пересечения медиан треугольника с центром вписанной в него окружности, перпендикулярна биссектрисе Z_C, то а + Ь +с _ 2ab 3 ~~ а+ Ъ 151. Стороны треугольника являются последовательными целыми числами, причем наименьшая сторона больше 3. Площадь также является целым числом. Доказать, что одна из высот треугольника делит его на два треугольника с целочисленными сторонами и что разность отрезков, на которые эта высота делит соответствующую сторону, равна четырем. 152. Пусть стороны треугольника равны а, Ь и с. Пусть та — медиана, 1а—биссектриса, опущенные на сторону а. Доказать, что (Ь + сУ <1та \2 *Ьс ^ \1а Г При каких условиях имеет место равенство? 153. Доказать, что для любого конечного множества точек на плоскости существует выпуклый n-угольник с вершинами в некоторых из этих точек, содержащий внутри себя остальные точки множества. 154. На плоскости дано конечное множество точек, причем все расстояния между точками множества различны. Каждая точка соединяется отрезком прямой с ближайшей к ней. Доказать, что в одной точке не может сходиться больше пяти построенных отрезков. 155. На плоскости расположено 1977 точек, никакие три из которых не лежат на одной прямой. Доказать, что из них можно выбрать 988 пар точек Av Вх; Л2, £2; ••• I ^988i ^988 так> чтобы отрезки, соединяющие точ- 26
ки Аг и В., i = 1, 2, ..., 988, пересекались не менее чем в 494 точках. 156. Доказать, что для любого натурального п на плоскости существует п точек, не лежащих на одной прямой, все расстояния между любыми двумя из которых выражаются целыми числами. 157. Доказать, что любые п точек на плоскости можно покрыть конечным числом кругов, сумма диаметров которых меньше п, а расстояние между каждыми двумя кругами больше 1. 158. Дано п городов, каждые два из которых соединены односторонним железнодорожным движением. Доказать, что существует гороД, р который из любого другого можно проехать, сделав не более одной пересадку. 159. Каждая из точек плоскости окрашена в один из трех цветов. Доказать, что найдутся две точки, окрашенные в один цвет, расстояние между которыми равно 1. 160. На плоскости расположено п точек, никакие три из которых не лежат на одной прямой. Доказать, что при п > 4 существует не менее С6 выпуклых четы- рехугольников с вершинами в этих точках. 161. На плоскости дано конечное множество точек, причем никакие три из них не лежат на одной прямой. Известно, что площадь произвольного треугольника с вершинами в этих точках не превосходит 1. Доказать, что все точки множества расположены внутри некоторого треугольника площадью 4. 162. Внутри выпуклого многоугольника расположено несколько непересекающихся кругов. Доказать, что многоугольник можно разбить на меньшие выпуклые многоугольники так, чтобы каждый из них содержал в точности один круг. (Советуем рассмотреть сначала случай равных кругов.) 27
163. Вписанный в окружность /г-угольник разбивается непересекающимися диагоналями на треугольники. Доказать, что сумма радиусов окружностей, вписанных в эти треугольники, не зависит от того, какие из диагоналей участвуют в этом разбиении. 164. Пусть точка О лежит внутри выпуклого 1000- угольника Av А2, ..., Л1000. Стороны 1000-угольника произвольным образом занумерованы числами 1, 2, ..., 1000; независимо от этой нумерации теми же числами занумерованы отрезки [ОА^у [ОЛ2], ..., [ОЛ10оо1- Можно ли это сделать так, чтобы сумма номеров сторон треугольников АгОА2у А2ОА3, ..., Ai00qOA1 была одна и та же? 165. Пусть М — выпуклый многоугольник. Обозначим через s минимальное число кругов радиуса 1, покрывающих /И, а через /—максимальное число попарно не пересекающихся кругов диаметра 1, центры которых принадлежат М. Доказать, что s<J. 166. а) В окружность вписан n-угольник, все стороны которого конгруэнтны. Доказать, что его углы также конгруэнтны. б) В окружность вписан /г-угольник, все углы которого конгруэнтны. Доказать, что если п нечетно, то его стороны также конгруэнтны. Верно ли это утверждение при четном /г? 167. Пусть ABCDE — выпуклый пятиугольник, вписанный в окружность радиуса 1, причем сторона АЕ — диаметр этой окружности. Пусть \АВ\ = а, \ВС\ = b, \CD\ = = с, \DE\ = d. Доказать, что а2 + Ь2 + с2 + d2 + abc + + bed < 4. 168. Доказать, что если плоский многоугольник имеет несколько осей симметрии, то они пересекаются в одной точке. 169. Окружности Sx и S2 пересекаются в точках А и В, причем дуга АВ окружности 52 делит площадь круга, 28
ограниченного Sv пополам. Доказать, что длина этой дуги больше диаметра окружности S1# 170. а) Доказать, что не существует многогранника с семью ребрами. б) Доказать, что для любого п^ 8 существует выпуклый многогранник, имеющий ровно п ребер. 171. В шахматном турнире участвовало два ученика VII класса и несколько учеников VIII класса. Каждый играл с каждым другим ровно один раз. Два семиклассника набрали вместе 8 очков, а все восьмиклассники набрали поровну очков. Сколько восьмиклассников участвовало в турнире? (При игре за победу дается 1 очко, за ничью —, за проигрыш 0 очков.) 172. Житель любого из городов Л, В, С знаком не более чем с одним из жителей каждого из других двух городов. Известно, что: а) число жителей А равно 6000; б) число жителей В, имеющих знакомых в городе С, не больше 2000; в) в городах В и С более половины жителей не имеют знакомых в А. Доказать, что число жителей городов Л, В и С, не имеющих знакомых в других городах, не меньше 1978. 173. После длительной разлуки встретились двое старых друзей. Один из них сообщил, что у него три сына, произведение возрастов которых равно 36, а сумма равна числу окон дома, около которого произошла встреча. Второй сказал, что он не может определить возраст детей. Тогда первый добавил, что его старший сын рыжий, после чего второй сразу же назвал возраст детей. Сколько лет было каждому сыну? 29
РЕШЕНИЯ 1. Заметим, что у первого члена сумма числителя и знаменателя равна 2, у двух следующих—3, у трех следующих— 4, у четырех следующих — 5 и т. д. Наше число стоит на 1917 месте в группе чисел с суммой числителя и знаменателя, равной 3894. Количество чисел до этой группы равно 1 + 2 + 3 + ... + 3892 = 1946- 3893 (с суммой 3893 в группе 3892 слагаемых). Отсюда п= 1946- 3892+ 1917. 2. Наименьшие неравные составные натуральные числа: 4, 6, 8, 9, .... Число 17 нельзя представить в виде суммы неравных составных натуральных чисел (17 < 4 + + 6 + 8), а любое нечетное 2k — 1, не меньшее 19, можно представить в виде 2fc —1 = 4 + 9 + (2k —Щ. Поскольку &^10, то 2k —14 четно и больше 4. 3. Обозначим через S2n сумму 30
Тогда 1 - + — — ... - = S2/2 — 2f— + — 4. Первое число k-и группы равно, очевидно, + 2 + ... + (k — Mi &-й группы равна: (1 + 2 + ... + (k — 1)) + 1 = M^zil 4- 1, а сумма k чисел Теперь о помощью индукции по п покажем, что S, +S3 +... + S2n_i=n*. При п = 1 имеем St = 1 = 1*. Далее, Sx + S3 + ... -f (2rt + 1)3 + (2rt + 1) + 1) = n* + 4n3 + 6n2 + An + 1 = (n + I)4. б. С помощью индукции нетрудно доказать, что fe = l 4 Используя эти формулы, имеем: 1? + 53 + 9^ + ... + (4п + I)3 = i (4k + I)3 = fc = 0 31
= у (43А3 _j- 3 • 42k2 + 3 • 4k + 1) = 43 У ft3 + 48 JQ&2 + 12 J o* + (n + 1) = + 8n+ 1). 6. Докажем индукцией по k, что я0 + #i + • • • +*к = = при 0 <1 k ^ /г — 1. При & = 0 доказывать нечего. Пусть наше утверждение верно для любого m^k. Тогда + x (х0 + 1 1 п — k п — k — 1 n-k— 1 Следовательно, наше утверждение верно и при k -f 1. В частности, при k = n — 1 получаем: *0+ *1 + ... + Хп-\ = 1. 7. Докажем с помощью индукции по &, что при При & = 2 имеем 23 — 7 = (— 2+ I)2. Пусть наше утверждение верно для любого k^.n, где п^2. Тогда (2/vh + Fn )a = (—2^„ — 4/7n«i + Fn )2 = (—F« - -4Fn_02 = ^ + 8Fn Fn-X ^ £ 32
2-1 -7F2n = 2(2Fn + Fn_x)* i -7f* = 2n+2 - т. е. наше утверждение верно и при k = n + 1. 8. Обозначим максимальное из возможных чисел через М(п), где п—число двоек, входящих в выражение. Докажем индукцией по п, что 2 .-2)) М (п) = 2<2( Наше утверждение тривиально при п =1,2. При п = 3 имеем 2 возможные расстановки скобок: 2<22> =16 и (22)2 = 16. Так что наше утверждение верно и при п = 3. Пусть теперь /1^4и скобки расставлены следующим образом: где все остальные скобки открываются и закрываются обе либо в нижних, либо в верхних квадратных скобках. Пусть в нижних квадратных скобках содержится k > 1 двоек. Тогда максимальное из выражений такого вида равно, очевидно, M(k)Min-k). По предположению индукции ( ••2)1\ M(k)M{n~k) = (22('"'2))) 2 = (2М1*~ 1))Щп"~k) Но, очевидно, так как M(k)^2, то М (k — 1) М (п — к) < М {k — 1)м<п ~ *> < М (п — 1). Следовательно, максимальное из чисел вида М (k)M(n - ® (k^l) не превосходит 2Щп-х\ т. е. УН (/ 2 Зак. 2126 33
По предположению индукции М (л — 1) = 2<2(2- *2)) (п - I двоек), откуда М (п) = 2м <« - П = 2(2<2'' '2)) (/г двоек), что и требовалось доказать. 9. Из предположения а162Сз>0, аа63с1>0, а361с2>0 получаем N = ахагаъЬфф&хсфъ > 0. Из предположения fc 0» — #2^3 > 0, —о^Ьфь > 0 получаем fc62^3CiC2C3>0. Но N=— M, значит, Л/>0 и —Л^>0, что невозможно. 10. Докажем индукцией по /г, что искомое число равно 10/г—9. При п= 1, очевидно, можно получить 1 = = 10-1 —9 квадрат. Нетрудно убедиться, что при /г = 2 можно выбрать 11 = 10 • 2—9 квадратов. Пусть утверждение верно для n = k, где k^2. Рассмотрим фигуру, состоящую из 8(£+ 1)—4 точек (рис. 4 для k = 3). • • \ч Л / ЧК ' ' ' / */ х/ / ; /У Рис. 4 34
По предположению индукции можно выбрать ровно 10fe—9 квадратов, ни одна из вершин которых не совпадает с точками Л, В, ..., Н. Рассмотрим теперь квадраты, одна из вершин которых совпадает с точкой Л. Нетрудно заметить, что найдутся лишь 3 таких квадрата: АВВгА19 ACEG, АС^Н. Таким образом, существует ровно 10 квадратов, по крайней мере одна из вершин которых совпадает с одной из точек А, В, ..., Н\ 4 квадрата, конгруэнтных AC^F^H, 4 квадрата, конгруэнтных АВВхАг, и квадраты ACEF, BDFH. Всего имеем 10£— 9 + 10 = = 10 (fe + 1) — 9 квадратов. 11. Прежде всего заметим, что после выполнения всех делений получим рациональное число вида г== где qv цг, ...,qn —исходные простые числа pv /7а,..., рп , переставленные каким-то образом. Так как все qt —простые, любая из таких дробей несократима и две. такие дроби равны тогда и только тогда, когда равны их числители (или знаменатели^). Теперь покажем, что можем получить в точности 2п~2 различных чисел. Рассмотрим число г. Предположим дополнительно, чт0 ?i < <7г < ... < Як и qk + 1 < qk + 2 < ... < qn . Назовем это число «хорошим», если оно получено из выражения рг: р2:...: рп при помощи расстановки скобок. Нетрудно убедиться, что если г «хорошее», то qx = pv qk+ i = /?2, т. е. r= Pigу. -ян РЯЯ' Докажем индукцией по k, что любое число такого вида является «хорошим». При п = 2 это очевидно. Пусть <72 = А + 1, где 2</</г. Тогда 35
= Pi . Г Pi Я*---Як I = РгРз- • -Pi— i " \Pi 4. \Qk +1 -• -Qn \ r== PiPl + tfs---Я* "" РРР -Pi 9k+f По предположению индукции число в квадратных скобках «хорошее», т. е. получается из выражения рг. р/+ь.. ,:рп при помощи расстановки скобок. Следовательно, и число г — «хорошее». Таким образом, «хороших» чисел ровно столько, сколько различных подмножеств {q2,.. .,<7л } множества {рз,/?4, •. ;Рп }• Легко убедиться в том, что таких подмножеств в точности 2п ~2. 12. Воспользуемся индукцией по п. Если п= 1, то имеем, очевидно, 2к наборов, причем для всех их, кроме набора (1, 1, ..., 1), т (с) = 0. Следовательно, сумма всех т (с) равна 1=1^. Пусть наше утверждение верно для любого р^п. Рассмотрим все такие наборы (alf a2, ..., ап), что 0 < а, <1 п + 1. Сумма т (с) по всем с = (alf a2, ..., ak), таким, что а, <1/г, равна по предположению индукции 1* + 2* + ... + nk . Рассмотрим теперь наборы с = (av a2, ..., afe), такие, что для некоторого i щ = п + 1. Пусть в наборе с = (av a2, . . ., ал ) в точности р а^ равно /2+1 и р < /г. Тогда т(с) = т (с'), где с" получается выбрасыванием из с всех а, , равных п + 1 (с' состоит из k—р чисел). Если зафиксировать места, на которых а, =п+1, то сумма т(с) по всем таким наборам равна сумме т(с') по всем наборам с' = (РР р2, .. ., Р^-р), таким, что 0<pt ^м. По предположению индукции эта сумма равна 1*- р + 2k~p + .". . + nk~ p. Но места, на которых а; =я+ 1, мы можем выбрать С£ способами. Следовательно, сумма т(с) по всем с = (av a2, ..., ап), таким, что ровно р (р<к) at равны п+1, равна 50 = Cpk(\k-p + 2k~p + ... + nk~p). Нерассмотренным 36
остался единственный набор с = (п + 1, п + 1» ..., п + 1), для которого т (с) = п + 1. Таким образом, искомая сумма равна: 1* + 2* + ... + п* + C[(lk- l + 2*~ 1 + ... + л*"') + + ... + п) + (п + 1) = (1* + Cil*"J + С? I*"2 + ... + * 1)+ (2 + 1) + 1 = О + 1)* + (2 + О* + ... + (л + 1)* + 1 = 1 + + 2^ + 3^ + ... + (п + l)k . 13. Рассмотрим множество М n-значных чисел, записанных с помощью цифр 1 и 2. Для каждого а £ М обозначим через М (а) множество Ь £ М, которые отличаются от а не более чем в одном разряде. Ясно, что в М (а) содержится в точности п отличных от а чисел, т. е. М (а) содержит п + 1 чисел. Пусть теперь av а2, ..., at — числа из М, такие, что при i Ф j числа щ и а,- различаются не менее чем в трех разрядах. Нетрудно убедиться з том, что М (at ){\М (а; ) = 0 при i Ф /. Поэтому множество М (ах) Q M (a2) (J ... [}M(at ) содержит ровно t (п + 1) чисел. Так как в М содержится 2п чисел, то t (п + 1) ^ 2п , откуда / ^ ——, что и требовалось дока- п + 1 зать. 14. Докажем, что хорды делят круг на т-\-п-\-\ частей. Доказывать будем методом математической индукции по числу хорд. Ясно, что одна хорда делит круг на две части и что в этом случае т = 0. Предположим, что для п — 1 хорд утверждение доказано. Пусть у нас есть п хорд в круге, пересекающихся в т точках внутри круга. Изъяв одну из хорд, получим п — 1 хорд и mx точек 37
пересечения. Эти п — 1 хорды по предположению индукции делят круг на п + Щ частей. Хорда, изъятая нами, пересекалась с остальными хордами в т—т1 точках. Она делится этими точками на т—тх + 1 частей, каждая из которых является границей между двумя частями круга, на которые он делится п хордами. Таким образом, после изъятия хорды 2{т—тх + 1) частей круга попарно объединились, образовав т—т1-\-\ частей. Следовательно, число частей, на которые круг делится п хордами, равно (п + т^ + (т —тх + 1) = п + т + 1, что и требовалось доказать. 15. Если уравнение не имеет решений, то условие выполнено. Пусть теперь уравнение имеет хотя бы одно решение п = п0. Покажем, что других решений нет. Если л<п0, то — > — и — > — , так как с > а, \ с ) \ с ] \ с I \ с ) с > 6. Отсюда -Г+(тГ>(тГ+(т)--1- atl + Ъп > сп . т^ ^ I а \п ^ ( а \п<> I Ь \п . Если п^>п0, то из неоавенств — < — , — < \ с ] \ с ) \ с ] ^ { Ь \п0 < — получаем: \ с I Следовательно, решения, отличного от п0у уравнение не имеет. 16. При п = 1 хг может быть произвольным. При п = 2 хг = х2 = 2. При п > 2 в качестве решения можно взять такой набор: хг= ... = хп _ 2 = 1, хп - i = 2, хп = п. 17. Заметам, что 1977 = 3 . 659 и число 659 простое. 38
Рассмотрим более общую задачу: покажем, что не существует таких натуральных х и у, что где р1 и р2 — различные простые числа. После возведения полученного равенства в квадрат имеем: Рассмотрим выражение (ргр2—х + у)2. Использовав (*), получаем: -* + У? = = 4 (Yy~(Yx~+ Так как рг и р2—простые числа, то это равенство может выполняться только тогда, когда у = №рхръ где k £ Z, Если £>1, тоУ*Г>/^р2, a Y^+Yy~> • Если k= 1, то у = ргр2у х = 0. Если & ±= 0, то У = 0, х = PiP2. Следовательно, уравнение во множестве натуральных чисел решений не имеет. 18. Так как 345 и 5у2 делятся на 5, то и 3#2 делится на 5, откуда следует, что х=5и, где и—целое число. Аналогично у = 3v для некоторого целого числа v. Уравнение принимает вид 5и2 + 3v2 = 23. Следовательно, 23 23 иа< —, г>2<—, откуда |и|<2, М<2. Перебором уста- 5 3 навливаем, что \и\ = 2, \v\ = 1. Отсюда получаем следующее множество решений: {(10, 3); (10, -3); (-10, 3); (-10, -3)}. 19. Ясно, что ни одно из натуральных чисел а, 6, с, d не может быть 1. Если хотя бы одно из них будет больше 2, то 39
Остается единственная возможность a=6 = o = d Это и есть решение уравнения. 20. Предположим, что такое равенство возможно! i_L + _L+ + ± а2 а22 а2п где ах < а2 < ... < ап. Из ах > 2 получаем поэтому что невозможно. 21. Если р = 2, то 22 + З2 = 13 =^= хту где х, т — натуральные, т>1. Пусть теперь р — нечетное простое число. Тогда + Зр = (2 + 3) (2Р"1 — 2Р-2 • 3 + 2р-3 . З2 —... + где А = 2р~1 —2р~2 - 3 + ... + З^-1 *. Заметим, что 3* = (5 — 2)k = bBk + (—2)k , где Bk —целое число (это следует, например, из формулы Ньютона). Следовательно, * Здесь мы воспользовались следующим легко проверяемым тождеством: ап + bn =(a + Ь) (а"-1 — а""2 Ъ + ап~ъЬ^ — . .. + Ьп"{)} где п нечетно. Ниже неоднократно будем его использовать, 40
А = 2"-1 — 2"-2 (5Вг — 2) + 2'-3 (5В2 + 22) —... — - (5Вр-! + 2*-*) = 5В + p2P-i, где В = — г*-2^ + 2*-3В2 —... + Вр-ь откуда 2р + Зр = 5 (5S + Р2Р-1) = 25 • В + Ър . 2^. Пусть 2р + Зр = хт, где х и т — натуральные, /л > 1. Тогда 25В поэтому х делится на 5. Но т> 1, следовательно, хт делится и на 25. Но при р=^=5 5р2'7-1 на 25 не делится. Если же р = 5, то 25 + Зб = 32 + 243 = 275 =£ х™ при т> 1. 22. Это уравнение имеет только одно решение: т = 2, л= 1. Пусть п четное. Тогда 3" = З2* = (8 + 1)* = 8/г + + Cl8k~l + ... + С\ - 8+ 1, откуда заключаем, что при делении на 8 32Л дает в остатке 1, а значит, 32Аг + 1 дает в остатке 2. При нечетном п > 1 32*+' = (8 + 1)* • 3, поэтому 32k+l+ 1 при делении на 8 дает в остатке 4. Отсюда заключаем, что равенство 2т = Зп -+■ 1 возможно разве что при т^2, это приведет к уравнениям 2—Зп = 1 и 22 — 3"= 1. Первое уравнение не имеет решений, а второе имеет единственное решение п=\. 23. Прежде всего заметим, что z — нечетное число и z не делится на 3. Любое число, не делящееся на 3, можно представить в виде 3k ± 1. Но (3k ± I)2 = 9k2 ± ± 6k + 1 = 3 (3k2 ± 2) + 1- Следовательно, z2 при делении на 3 дает в остатке 1. Легко заметить, что при четных х 2х при делении на 3 дает в остатке 1, а при нечетных х—дает в остатке 2. Так как остатки при делении на 3 в правой и левой частях уравнения должны совпадать, то х — четное число.
Пусть х = 2s. Уравнение принимает вид 4* + ЗУ = z*. Так как г — нечетное число, a (2t + I)* = \t (t -f 1) •+- 1, то z2 при делении на 4 дает в остатке 1. С другой стороны, легко заметить, что Зу при делении на 4 дает в остатке 1 при четных у и 3 при нечетных у. Следовательно, у четно. Пусть у = 2q. Уравнение принимает вид 4* + 9* = г2. Откуда 22* = z2— (3*)2, 22s=(z— 3*)(r+3'). Таким образом, z — 3" = 2* , 2 + 3? = 2Ь, а + 6 = 2^ тогда 2г = (z — &) + (г + 3*) = 2а + 2Ь = 2* (1 -f 2*-*) (очевидно, а < 6). Так как числа 2 и 1 + 2*-° нечетны, то а =* 1, Ь = 25 — — 1, z=l + 225-1 = 1 + 2Х-К Отсюда Следовательно, (2 + 3*) — (г — 3«) = 2*-» —2, 2 • 3* = 2 (2*-2 — 1) или 3? = 22<s-o _ 1, 3^ = 4s—1 — 1. Число 4s-1 — 1 при делении на 4 дает в остатке 3, следовательно, д—нечетное число (см. выше). Но тогда 42
3* + 1 = (3 + 1) (3*-1 — З*-2 + ... + 1), откуда 4—1 = 4S, где S = З*-1 —З^2 + ... + 1 —нечетное число (так как S—сумма нечетного числа нечетных слагаемых). Следовательно, S = 1, s = 2, откуда х = 25 = 4, 3* = 2*-2 _ 1 = 3, q = 1, у = 2<7 = 2, ^ = 24 + 32=25, z= ±5. Итак, во множестве целых чисел уравнение имеет следующее решение: х = 4, у = 2, г = ± 5. 24. Умножим обе части уравнения на 4 и прибавим к обеим частям по 1. Уравнение примет вид 4#2 + Ау + 1 = 4*4 + 4х3 + 4*2 + Ах + 1 или (2# + I)2 = 4*4 + 4jt> + 4л:2 + 4л: + 1. Заметим, что 4х4 + 4л:3 + 4л:2 + 4л: + 1 = (2*2 + х)2 + Зх2 + Ах + 1 = = (2л;2 + *)2 + (3*+1М*+1). Так как при я< — 1 и я>0 (Зл: + 1) (х + 1) > 0, то при х < — 1 и л; > О 4л^ + 4*3 + 4*2 + 4л: + 1 > (2*2 + л:)2. С другой стороны, Ах* + Ахъ + Ах2 + Ах + 1 = (2х2 + х + I)2 — х2 + 2х = = (2х2 + х + I)2 — (х — 2)х; следовательно, при л:<0 и л:>2 Ах* + Ахъ + Ах2 + Ах + 1 < (2х2 + х + I)2. Таким образом, при х < — 1 и х > 2 43
(2х2 + х)2 < 4*4 + 4r* + 4*2 + Ax + 1 < (2*2 + x+ I)2. Поскольку 2я* + л: и 2х2 + * + 1 — последовательные целые числа, то между (2*2 + х)2 и (2*2 + л: + I)2 не может лежать квадрат целого числа. Следовательно, при*< — 1 и х > 2 уравнение не имеет решений в целых числах. Осталось рассмотреть четыре случая: а) х= —1, б) х =* = 0, в) х= 1, г) х = 2. а) * = — 1. Тогда У2 + У = у(у + 1) = 0; у = 0, у = = — 1. б) л: = 0. Тогда у (у + 1) = 0. Следовательно, у = 0 или у = — 1. в) *= 1. Тогда у(у+ 1) = 4. Так как полученное уравнение во множестве целых чисел решения не имеет, то при х = 1 решений нет. г) х = 2. Тогда у2 + у = 30, откуда у = — 6 или у =5. Итак, получили следующее множество решений: {(-1; 0), (-1; -1), (0; 0), (0; -1), (2; 5), (2; -6)}. 25. Предположим, что x^y^z. Тогда 4х делит 4^ и 42, а следовательно, и и2. Так как 4х = (2х )2, то — =v2, где и—целое число. Разделив данное уравнение на 4Л , получим: 1 + №~х + 4*-* = v2. Заметим, что в левой части этого уравнения стоит четное число лишь тогда, когда х = у, z > х или у > х, г = х. Так как последнее невозможно, то в левой части четное число может быть лишь в случае х = у, z >*. Тогда 2 4- 4*-* = v2. Следовательно, v = 2t—четное число. Но тогда v2 делится на 4, а 2 + 4Z~* не делится. Получили противоречие 44
Следовательно, число 1 + ¥~х + 42~Л = ь2 — нечетно, значит, v тоже нечетное число. Представим v в виде v = 1 + 2т/, где / — нечетное число. Тогда 1 + Щ1-х _)_ 4*-* = (1 + 2mt)2 или 42~* = 2 • 24 + (2"Ч)2, 42~* = 2т+!/ (1 + 2т~Ч). Вынесем в левой части за скобку Ау—Х. 42-^) = 2т+Ч {2т~Н+ 1), + 42-*0 = 2m+V (г^1/ + 1). Число 1 + 42~^ четно лишь в случае z = у, тогда 2т+Ч (2т-Н + 1). Поскольку t нечетно, то t = 1, т. е. при 2 = у t = 1, отсюда 22t/-2*+i = 2^+1 (2^-i + 1). При m> I 2™-1 + 1 —нечетное число, большее 1. Значит, т=1. Следовательно, 22у-2х+{ = 23, откуда 2у — — 2д: Н- 1 = 3 и y = z = jc+l. Но 4* + 4*+* + 4*+» = = (2х • З)2, т. е. при любом *>0 (л:, л; + 1, jc+ 1, 3 • 2х ) — решение данного уравнения. Пусть теперь г > у, тогда число 1 + 42-^ нечетно, и из равенства + 42-*) = 2т+Ч (1 + 2т-Ч) следует, что при /я> 1, 2у — 2д; = т+ 1, 45
— t2), 2~2H-*+i _ f). Но, если последнее выражение не равно нулю, получим противоречие, так как в этом случае t — 1 < t < г2-2****1 + t, тем более t _ 1 < 4*-*-1 (22-2<>+*+i + t) (22-*/+*+l — О, поскольку 4у-х-1 (22-2а/+*+1 —/) = 22-*-1 — 4У-Х-Ц — натуральное число. Следовательно, ^у-х-1 (2г-2у+х+\ ^ ^ (2г~2у+х+1 А, = Q Но, очевидно, 4^-^!>0, г*-2^^1 + / > 0. Следовательно, Но ^ нечетно. Следовательно, /=1 и г — 2# + *+ 1 = 0, г = 2у — х+1. С другой стороны, 4^ -j- 4^ + 42^-*-1 = (2х + г2^-^-1)2. Таким образом, любая четверка целых чисел (я, у, 2jc —«/ — 1,2* + 22^-*-i), где *> 0, у > 0, 2х —у — 1 >0, является решением данного уравнения. Заметим также, что при у •= * + 1, 2у — * — 1 = х + 1, т. е. среди решений вида (х, у, 2х—уу 2х + г2^"1) содержатся и все решения вида (я, х-\- 1, х + 1, 3 • 2х), полученные раньше. Случай т= 1, #<*/<£ оставляем читателю. Итак, данное уравнение имеет следующие решения во множестве целых чисел: {(х, у, 2х — у — 1, 2х +2*<-*-1) |xeZ, y€Z t/>0, 2л: — у— 1>0}. 46
26. Проделаем следующие преобразования данного уравнения: х (х*- у*У -х = у(х* -у*у (х-у)х = х+у х—у (х — у) х ={х * Заметим, что из условия следует, что х—#>0, следовательно, ln(x—y)*-v = \n{x + y)*+v9 lnz __ \nt т~~т* где z = x— yy i = x + у. Рассмотрим функцию f(t) = —. Найдем ее производную по - ^. Она обращается в 0 в точке t = г, при 0 < / < г /'(/) > О, при t>e Г(0<0. Следовательно, при 0</<е /(0 строго возрастает, а при £ > е строго убывает. Следовательно, если /(/j) = /(^2) и tx < /2, то 0 < t± < б, /2 > е. Возвраща- 47
ясь к нашему уравнению, видим, что так как z = * — — У <х + у = t9 то г<е<§. Ног —натуральное числр. Следовательно, либо а) х — у = 1, либо б) х —у = 2. а) Если х — у = 1, то откуда х + у = 1 и j/ = 0. Получили противоречие, б) Если х—у = 2, то 2*+*> = (х + у)2. Но при л > 4 2" > л2. Действительно, 25 = 32 > 52 = 26. Пусть наше утверждение верно при 5<;&<!/г. Тогда 2n+i = 2 . 2п > 2п2 >(л + I)2, поскольку при п >5 Следовательно, 2 < х + у ^ 4. Непосредственная проверка показывает, что х + у = 3 не подходит, а х + у = 4 подходит. Следовательно, х —у = 2, # + у=4, откуда * = 3, у=1. 27. а) Умножив обе части уравнения на 4 и прибавив к обеим частям по f/4, получим: У4 = 4л:4 или 4^"+1 + ух = (2л:2 + у2)2. Рассмотрим отдельно четыре случая: 1) m = 0. Тогда Но так как jc и у — целые числа, то при |*| > 1 х2(х2 4- + f/2) > f/2 и у2 > у. Следовательно, *2 (л:2 + у2) > у. Таким образом, в этом случае обязательно х = 0, откуда и у = 0. 2) m = 1. Тогда 48
следовательно, у2 + 4 = и2, где и —целое число, или (и— У)(и + у) = 4. Пусть 0 <; у < м. Тогда, если м — у > 2, то а + у > 2, и2 — У2 > 4. Пусть а — у = 2. Тогда и + у = 2, откуда у = 0, и = 2. Если же а — у = 1, то м + У = 4, откуда 2у = 3. Получили противоречие. Следовательно, в этом случае у = 0, откуда и я = 0. 3) т = 2. Тогда откуда у2 -+■ 4у = а2, где и—целое число, или (у + 2)2— 4 = а2. Значит (см. случай 2)), и = 0,у+2=±2и либо # = 0, л: = 0, либо у = — 4. Тогда *2(*2+ 16) = —64, но х2(х2-\- 16) >0. Следовательно, и в этом случае единственное решение х = 0, у = 0. 4) т > 3. Тогда откуда 1 + Аут-Ъ = м2, где и — целое число. Так как 1 + 4ут—з — нечетное число, то и и—нечетное. Пусть и = 2v -f 1. Тогда 1 + \ут~ъ = (2v + I)2, откуда Ьут-Ъ = 4и (V + 1) или ут-Ъ = v (V _j_ 1^ 49
Так как произведение двух последовательных целых чисел не может равняться 1, то т>3. С другой стороны, если т — 3>1, то v(v+ 1) = Уа , где а = т — 3>1. Но числа v и v + 1 взаимно просты. (Действительно, если d делит v и v+ 1, то d делит и (v+ l)—v= 1, откуда d = 1.) Следовательно, v = r° , v + 1 = s° , где (s, г) = 1 и а > 1. Но тогда 5* — Га = 1 И Sa _ га = (s _ Г) (Sa-1 + ^-2Г + . . . + ^-1), откуда s — г = 1 и s0""1 + sa~2r+ ... +/41""1 = 1. Левая часть последнего равенства содержит по крайней мере два члена, так как а> 1. Следовательно, г = 0, откуда v = 0 и у = х = 0. Таким образом, при т > 4 единственное решение х = у = 0. Пусть теперь т = 4, тогда у - и (и + 1). Следовательно, (2*2 + у2)2 = у4 (2v + I)2, откуда 2х2 + v2 (v + I)2 = ± v2 (v + I)2 (2v + 1). Если 2v + 1 > 0 или и > 0, то 2*2 + v2 (v + I)2 = v2 (v + I)2 + 2у3 (у + I)2, откуда X2 = VV2 • (V+ I)2. Значит, v = t2, где t—целое. Следовательно, x2=t*(t2-\~ + I)2, откуда х = f (t2 +1), у = t2 (t2 + 1). С другой стороны, [^ (t2 + I)]2 {[*3 (t2 + I)]2 + [t* (t2 + I)]2} = t" (t2 + + l)4('2+l) = l'2('2 + l)]5, т. e. 50
при любом целом / числа х = ts (t2 + 1) и у = t2 (t2 + 1) удовлетворяют условию задачи. Таким образом, при т=4 все решения исчерпываются множеством {х = t3 (t2 + 1), y = t2(t2+\)\ttZ}. б) Умножив обе части уравнения на 4 и прибавив к обеим частям по у2, приведем уравнение к виду (2л:2 + у)2 = у2 + 4ут+[. Разберем отдельно четыре случая: 1) т = 0. Тогда (2х2 + у)2 = у2 + 4у. Следовательно (см. а), случай 3)), (у + 2)2=4, 2х2+у=0. Следовательно, г/ + 2 = ±2, т. е. либо у = 0, тогда и х = 0, либо у = —4. Но в последнем случае имеем 2х2 — 4 = 0 или х2 — 2 = 0, а это уравнение не имеет решений в целых числах. Таким образом, при т = 0 х = у = 0. 2) т>2. Тогда Число 1 + 4t/m~1 нечетное. Пусть 1 + 4ут~х = (2v 4- I)2, тогда 4ут"х = Av(v+ 1), но тогда (см. а), случай 4)) у = 0, х = 0. 3) т = 1. Тогда (2х2 + у)2 = 5у2, но 5, не является квадратом целого числа, следовательно, у = х = 0. 4) т = 2. Тогда (2х2 + */)2 = #2 0 + 4*/), откуда 1 + 4у = (2о + I)2, где v — целое число, v > 0; следовательно, 0 = 0(0+1). Подставляя в исходное уравнение, получим: 51
[2х2 + v (v + I)]2 = v2 (v + I)2 (2v + I)2 или 2x2 + v(v + 1) = ± v(v + 1) (2v + 1), но левая часть всегда положительна, следовательно, знак минус можно отбросить. Тогда v(v+ l)(2o+ l) = 2v*(v+ l) + v(v+ 1), откуда x* = v2(v+ 1). Следовательно, v + 1 = /2, где ^ —целое число. Тогда С другой стороны, т. е. любая пара чисел х = t (t2 — 1), y = t2(t2— 1) является решением уравнения. Ответ: а) При т Ф 4 единственное решение л: = = 0 = 0; при /72 = 4 x = ^(^2+l), y = t2(t2+ 1), где /— произвольное целое число. б) При тф2 единственное решение х = у = 0; при /п = 2 х = /(/2-1), у = t2(t2 — 1), где ^ — произвольное целое число. 28. Предположим противное: пусть {х, у, z, t) — решение, у которого / — минимальное из возможных. Условимся через (аь Oj, ...,ak) обозначить наибольший общий делитель чисел alt а2, ..., ak. Тогда (х, у, z, t)=l. Действительно, если (х, у, г, t) = k, то х = xYk, у = yxky z = zxk, t=^txk. И очевидно, {xlf ylf zly t±)—решение данной системы. Тогда, если А>Ь то tx<t—противоречие. 52
Положим а = (х, г), Ь = (у, t), тогда х = ар, у = bq, z = asy t = W, причем (p, s) = (q, /)=»!. Заметим также, что (а, 6) = (xy у у Zy t) = 1. Так как *t/ = ztf то ap&<7 = = asbl. Так как (р, s) = (9, /) = 1, то р = /, <7 = s, т. е. найдутся такие натуральные а, 6, р, ^, что л: = ар, у = bqt z = aq, t = bp и (a, ft) = (p, q) = 1. Тогда, поскольку x2 + у2 = z2 —t2, то a2p2 + b2q2 = a2q2 — b2p2y a2p2 + b2q2 + a2q2 + b2p2 = 2a2q2t {a2 + b2) (p2 + q2) = 2a2q2. Заметим, что так как (a, b) = 1, то (a2, b2) = 1 и (a2 -f + b2, b2)= 1. Аналогично (p2 + #2, #2) = 1. Следовательно, имеются две возможности: а) а2 + b2 = 2q2t р2 + q2 = a2; б) а2 + b2 = q2y p2 + q2 = 2a2. Рассмотрим случай а). Заметим, что если р и q нечетны, то р2 + q2 четно, но не делится на 4, а а2 либо нечетно, либо делится на 4. Следовательно (так как (р, q)= = 1), а нечетно, и одно из чисел р, q четно, а другое нечетно. Если q четно, то 2q2 = а2 + Ь2 делится на 8. Но а нечетно, откуда b нечетно и а2 + Ъ2 не делится на 4. Получили противоречие. Следовательно, в случае а) числа а и q нечетны, р четно. Так как а2 — q2 = р2 = (а — q) (a + q), то p = kmnt a + q = km2y a—q = kn2 для некоторых натуральных ky ту п. Тогда 2a = k(m2 + n2)y 2q = k (m2 — /г2), p=kmn. Но (2р, q) = 2, откуда либо k= 1, либо k = 2. Если /г = 1, то 2а = т2 + п2 —четное число. Следовательно, так как а нечетно, тип тоже нечетны. Но тогда и р = тп нечетно. Итак, k = 2, а = т2 + п2, q = т2 —/г2, р = 2тп. Но тогда а2 + Ь* = (т2 + п2)2 + Ь2, 2q2 = 2(m2—n2)2, (т2 + п2)2 + Ь2 = 2(т2— п2)\ 53
b2 = т* + n* —6rn2n\ &=</* —р2 или 62 + Р2 = 92. Так как р четно и (q, р) = !, то, рассуждая так же, получаем, что Ъ = г2—s2, <7 = т2 + я2 = = г2 —sa, р = 2rs = 2т/г, т. е. (т, /г, г, s) —решение данной системы. Но s<Cp^t, а решение (х, у, г, t) выбрано так, что / минимально. Получили противоречие. Случай б) разбирается аналогично. 29. Используя тождество {х} = х — \х], запишем уравнение Xs — [а:] = 3 в виде хъ — (х — {х}) = 3 или хъ — х = = 3— {*}. Поскольку 0<{*}<1, то 2<г*— х<3. При д:<—1 х2 — 1>0, хъ— х = х(х2 — 1)<0. При —1<л:<0 хъ— х = х(х2 — 1)<— х< 1. При 0<л:<1 хъ — x<x3<l. При х> 2 *з—я = *(.*;* — 1)>2(22 — 1) = 6. Таким образом, 1 < х < 2. Следовательно, М = 1 и данное уравнение превращается в уравнение хъ —1 = 3, которое имеет следующее решение х = у 4 . 30. Рассмотрим х, заключенные в интервале п^х< <л+ 1, где /г—неотрицательное целое число. Для всех таких х [х]2 = /г2. Для того чтобы выполнялось равенство, левая часть уравнения тоже должна быть равна п2, поэтому х < У п2 + 1 . Таким образом, х лежит в одном из интервалов [п, V п2 + 1 [, где л = 0, 1, 2, ... . 31. Из равенства х = [х] + {х} получаем: {х2} = {(М + W)2} = {М2 + 2 [х] {х} + {х}*} = = {2 М {*} + {*)■}. Отсюда заключаем, что равенство {я2} = {х}2 возможно лишь тогда, когда 2[х]{л;}—целое число. Поэтому 0 <х < 1 или х = т + -т^, где т —любое целое число, а 0 < / < 2т.
32. Сгруппируем слагаемые так, чтобы у первого слагаемого группы под знаком корня был квадрат какого-то числа р, а у последнего слагаемого группы число, стоящее под знаком корня, было равно (р + I)2 — 1, т. е. s = ([УТ] + iVj] + гУ~з~}> + «УТ] + [)/"5"] + + [/ 6] +[У 7] +ц/" 8 ]) + ... +([У>-1)2] + + [У (л -1)2 + 1 ] + ...+ [У"2- И). Таких групп будет п — 1 (у 1-й группы на первом месте \й, у 2-й на первом месте 22, у последней — (п — I)2). В р-й группе 2р + 1 слагаемых ((р + I)2 — 1 — (^ — 1) = «= 2р + 1). Каждое слагаемое р-й группы равно первому слагаемому, т. е. р. Тогда s=sV(2p+i)= ? ? р=1 _ о (/г 1)п(2а— 1) 6 ^2 "" ' 6 ""' 33. Представим а в виде а = [а] + {а}. Тогда = [М+М]. Но 0<{се}<1, поэтому [«] , {«}! _ Г[«11 - + -J - [-J- 34. Выберем такое натуральное k, что I -\ i х<С1 + —, где / — некоторое целое число. Тогда [х] + [дс 4- 4"] + ». + [« +iLJr*] = («-*+ 1) М — 1. 55
Отсюда 35. Пусть при каких-то xlt х2(х1^>х2) Ь \ ( ах9+ b ) } (^}тогда ах1±Ь — x2) tu tn tn не может быть целым, так как ант взаимно т просты, а хх — х2<т, поскольку О^х^^т—1, 0^х2^т — 1. Следовательно, |а* 1 принимает т различных значений. Эти значения суть правильные дроби со знаменателем т: —, —, —, ... , т~~ . - О-4- — 4- — 4- т—1 _ х=о i - > т т '" т "" 1/1(О| (/ - 1ЧЧ 1 (т—1) т т—1 = ~^Г \* ~Т *• \" ••• г \Щ — Ч) == ~ZT ' о == —о—• 36. Можно считать, что аир удовлетворяют неравенствам 0<.а<1, 0<р<1. В самом деле, пусть а = = а7 + л, р = р7 + т, где т и л —целые числа, а 0<а7<1, 0<;р'<1. Тогда, воспользовавшись очевидной формулой [х + л] = [л:] + л, справедливой для любого х и целого л, получим: [2а] + [2р] = [2а7 + 2л] + [2р' + 2т] = [2а7] + [2р7] + + 2т + 2л; [а] + [а + р] + [р] = [а7 + л] + [а7 + р7 + + m + п] + [р7 + т] = [а7] + [а7 + р7] + [р7] + 2т + 2л, т. е. задача свелась к числам а' и р7 из единичного 56
интервала. Далее, [а] = [р] = 0, поэтому осталось доказать неравенство Если [а + |3] = 0, то неравенство очевидно, если [а + Р1 = = 1, то а + р> 1, поэтому или а, или р не меньше -у. Тогда или [2а] = 1, или [2р] = 1. Неравенство доказано и в этом случае. 37. Заменив в неравенстве х на [х] + {#}, получим: Так как {х} — 1 < 0, то отсюда следует, что [х] > 1. Следовательно, х^2. 38. Воспользуемся неравенством \ \ fLi^L_^L_i— — — 4- — — а{ + ь{ > 2a'bl — о &i •" b2 ~~ bi "■ _Oi_ ~~ bx "" flj ~~ a^ -^ a^j ~~ " bx 39. Обозначим через ^ произведение т последовательных чисел, начиная с ait расположенных по часовой стрелке. По теореме о среднем арифметическом и среднем геометрическом имеем: Перемножая эти неравенства (i = 1, 2, ..., п)% получим: \п > Voxc% ... сп. Но легко заметить, что число щ входит сомножителем ровно в т различных чисел с,- (один раз на первом месте, один раз на втором и т. д.). Следовательно, ^2 .. Сп = а?п2 д, откуда 57
или Равенство — = -/ q возможно лишь тогда, когда а, = a/+i = ... =а/+т_!. Следовательно, &!&2 ... &« =• = тпа1а2 ... ап тогда и только тогда, когда а1 = а2=я = ... =ая. 40. Положим Л = (х —а)2, В = (х + яа)2. По теореме о среднем арифметическом и среднем геометрическом имеем: ~ л + 1 Но п 1 2п 2 л+1 п+\ n+i = х2 + яа2. Следовательно, 2 [(* — а)" (* + /га)]/г+1 < х2 + /га2. Докажем, что равенство невозможно. По теореме о среднем арифметическом и среднем геометрическом равенство имеет место тогда и только тогда, когда А = В, т. е. (х—а)2 = (х + па)2. Решая это уравнение, получим х = а( ~п \ но это невозможно. Действительно, а>О, /z^l, откуда следует х ^ 0, но по условию х > 0. 58
Таким образом, полученное неравенство является строгим: (х — а)»(х + па)<(х2.+ па*) 2 . 41. Прежде всего докажем, что W не может быть меньше числа пУ N . Действительно, если W, Blt В2, ,.., Вп удовлетворяют условию задачи, то по теореме о среднем арифметическом и среднем геометрическом имеем: Откуда следует, что W^/гУ N. Заметим, что если число пУN нецелое, то W ^ [п У N ] + 1, поскольку W целое. Рассмотрим сначала случай, когда пУ N —число целое. Тогда У N —число рациональное и, следовательно, является целым числом1. Положим Вг = В2= .. . = = Вп = У1Т. Тогда Вг + В2+ ... +Вп = пУ~Ы и В^ъ ... Вп = N. Таким образом, если число nY N — целое, то W = п У N . Случай, когда пУ N — нецелое, значительно сложнее. Обозначим через М число [пУ~7Г] + + 1. Как мы знаем, W не может быть меньше М. Заметим, что так как пУ N не является целым, то м> 1. Представим М в виде M = kn-\-r, где k — целое число, 0^/-</i. Рассмотрим два случая: a) k > 1. Положим В± = В2= ... = Bn-i = k, Bn = — k + г. Тогда П j П 1 Действительно, если у N =— и дробь несократима, то Рп N = —тг — целое число, и, так как р и q взаимно просты, то ^=1, т. е. yY=pZZ. 59
В± + В2 + ... + Вп = k (п — 1) + k + г = kn + г - М9 В±В2 ... Вп = kn~~l (k + г) ^ так как А2 — k = k{k — 1) > 1. Но л■ = kn + г, откуда i/^N ^А + -^-<А+ 1, так как /• <л. Следовательно, BiB2 ... Вп > -j/ Af, и в этом случае W =» = М = [л^ЛГ]+1. б) А = 1, г > 0, т. е. М = п + г, где 0 < г < п. Положим Вх = В2= ... = Вг = 2, Br+i = ... = Вп = 1. Тогда < М = п + г, откуда \/~ N <1 + -^- < г + 1. Индукцией по г легко доказать, что при любом натуральном г 2Г ^ г + 1. Действительно, при г = 1 2Х^ 1 + 1. Пусть неравенство верно при г^^. Тогда Таким образом, В^ ... Вп = 2'> г + 1 > ^"ЛГ. Мы видим, что в этом случае W = M = [nj/~N]+ 1. в) k = 1, г = 0, т. е. М = д. Тогда п = М > л ^"^ откуда >/~N < 1, чего не может быть. г) k = 0, т. е. УИ<л, тогда л>Л1>п>^'лГ, откуда ^"/V7 < 1. Получили противоречие. Таким образом, W = пYN» если число nY~N~ целое, и W = [nYN] + 1, если число nyfN нецелое. 42. Оба неравенства доказываются на основании следующего утверждения: если f—такая функция, что для любых х19 х2£]а\ Ь[ 60
/(*,) 2 то для любого п и произвольных хг, хъ ..., хп £ ]а\ Ь[ /(*i)+ /(*«) + ... +/(*„) Докажем это утверждение с помощью индукции по п. Пусть при п = k оно справедливо. Рассмотрим сумму где хь а:2, ..., xk+i e ]«; Ь[. Тогда, очевидно, число £-:r-j =ii также принадлежит ]а\ Ь[. В каждой из двух скобок в написанной выше сумме находится k слагаемых. Применяя предположение индукции, получаем: Далее, ^ of / ^ + ^2 В результате )' 2ft получаем I f (x W -^ /'f I*1 "^ е1 + *,+...+ж/и.,^ 2ft ft + 1 неравенство м 2ft (ft + )' xk+\ . ft )(*-!) 1) 61
откуда вытекает требуемо©! Для доказательства неравенства а) задачи достаточно применить доказанное утверждение к функции sinx, х € ] 0; -~ [. Для доказательства 6) применим это утверждение к функции log sin л:. Получим при а£ £ ]0; -^-[ log sin ax + log sin cxg + ... + log sin an <1 откуда следует нужное неравенство. 43. Покажем сначала, что для любого х хотя бы одно из чисел | sin x |, | sin {x+ 1)| больше ~ . о Рассмотрим единичную окружность и прямые у= _L ! 3 и у = — — . Пусть Л, В — точки пересечения этих прямых с единичной окружностью (рис. 5). Достаточно, очевидно, пока- /\ зать, что АОВ меньше 1 радиана. В самом деле, Рис. 5 sin АОВ = sin (2 СОВ) = sin (2 arc sin -^) = 62
2 sin (arc sin -J-J ( i i) * cos (arc sin i-) = .*. у l - \- sinJ ^ 07 0 < AOB Так как sin 1 > sin-J- = -^~- >0,7 и 0 < AOB< -5-, то ЛОВ< 1. Теперь можем доказать требуемое неравенство: sin л | , 1 sin (л+1)1^ , /| sin (л+ 2)| , |sin(/z + 3) п~ + 1Г+1 ) + \ JT+2 I ^+3 |sin(3n-2)[ |sin(3n— 1) |sinyi| + lsin(n + l)| , ] sin (n +- 2) 1 + I sin (n + 3) I |sin(3n —2)l + |sin(3n—1)| ^ Ъп— 1 > ~ # 3/1—1 ^ "9JT "" "9"* 44. Докажем сначала, что для любого натурального п 3"^/г3. Будем доказывать это утверждение индукцией по п. При /г=1, 2, 3, 4 имеем Зх>13, 32>23, З3 ^ З3, З4 ^ 43. Пусть наше утверждение верно при п = k, где k ^ 4. Тогда по предположению индукции 3*+i = 3 • 3* > 3 • /г3 = kz + 3k2 + 3k + (k — 3) k2 + + (k2-3)k>(k+l)\ так как при /г>3 (k — 3)^2>1, (fe"—3)A>1. Итак, для любого натурального п Зп > /г3 или -\f 3 ^ у^ /г . 63
Пусть т и п—натуральные числа и п^.т. Тогда у п <У п <у 3 , что и требовалось доказать. 45. Покажем сначала, что fl +-7H <3 для любого п ^ 2. В самом деле, ~ 3! n3 ~ '•• ~ пл ' 2!n ~ (Ai—l)(n —2) _L^9_i J_i J_ . ~* 3!n2 "*" '" "*" nn ^ "^ 2 ' 22 "*" •'• Здесь мы воспользовались тем, что для любых /1^2 и I — 1) (П — 2) ... (Al — 6) ^ 1 ^ 1 (^ + 1) !ахл ^ (Л+1)» "" 2^ " Пользуясь доказанным выше, получаем: - v Итак, получили: V [ п } п п(м—о я + l 1. Ь
46. Раскроем скобки: , ab =aui a* _i_ bi л. 2ab ' cin n rv-\c n ' г»лс п *^ ein ГУ. sin a cos a * cos a * sin a n sin 2a" Поскольку 0<sina<l, 0<cosa<l, 0<sin2a<l, то -r-5— > 1, > 1, . о > 1, откуда и получаем sin a ^ l cos a ^ ' sin 2a-^ ' J J требуемое неравенство. 47. Положим S = * + *»+д" +^t тогда xl) — nS2 = 0. Заметим, что условию Следовательно, = [x] - 2xxS + S2) + (x] - 2^2S + S2) + ... + + {x*n—2xnS + S2) = (^ -S)2 + (jc2 - S)2 + ... + Так как из сц + al + ... + ап = 0 следует, что ax = flj =з ...= ал = 0, то ^! = л:2 = ... = л:„ = S. 48. Рассмотрим сначала случай, когда по крайней мере одно из чисел а—Ь, а— с, b —с равно 0. Пусть, например, а ~Ь = 0, тогда 0 = (a — b)2 < Q2 + ^2 4- с2^ так как сум_ ма квадратов всегда неотрицательная величина. Пусть 3 Зак. 2126 65
теперь а — ЬфО, а — сфО, b—сфО. Обозначим через т наименьшее из чисел \а — Ь\, \а—о\, \Ь—с\. Пусть для определенности а < b < с. Тогда по определению b — а^т, с — б^т, откуда с — а= (с — Ь) + Ф — а)^ ^2т. Следовательно, (а—Ъ)2 + (а —с)2 + ф — с)2^т2+4т2 + т2 = 6т2. С другой стороны, (а —Ь)2 + (а—с)2 + ф—с)2 = 2(а2 + Ь2 + с2) — — 2 (ab + ас + be) = 3(a2 + b2 + с2) — (а2 + Ь2 + с2 + + 2 (ab + ас + be)) = 3 (а2 + Ь2 + с2) — (а + b + с)2. Так как (а + b + с)2 ^ 0, то, сравнивая с предыдущим неравенством, получим: 6т2 < (а — Ь)* + (а —с)2 + ф —с)2 = 3(а2 + Ь2 + с2) - — (а + b + с)2 < 3 (а2 + Ь2 + с2) или т ^ 2 Нот2—одно из чисел (наименьшее) (а — б)2, (а—с)2, ф—с)2, т. е. наименьшее из чисел (а — б)2, (а — с)2, а2 -Ь Ъ2 4- с2 ф — с)2 не превосходит —^—!—, а это и нужно было доказать. Замечание. Заметим, что равенство возможно лишь когда b — а = т, с — b = т, с—а = 2m, b + с + а = 0. Но тогда b = а + т, с = а + 2m, a + b + с = 3 (а+т)=0, откуда а = —т. Следовательно, равенство возможно лишь когда а = —т, 6 = 0, с = т, где т —произвольное положительное число. (Напомним, что мы предполагаем а<Ь<с.) 49. По условию S2 = T±r[ l(a1— а2)2 + (а2— аг)2 + ... + (ая_, — ап)2}. 66
Это равенство можно записать следующим образом: ^-тЧ 2 (о, -ai)\ п~ 4<i<i<n где знак 2 означает, что мы суммируем по всем па- \1' } рам таких индексов i и /, что 1 <; i < / <; /г. Тогда 52 = -^2 (2<«-<*)•)< -^тт 2 (2(«/ -«i) n X i=2 V = l ^^ «=2 V/=l так как для любого / (щ—а/)2^(а£ — а^2. Следовательно, поскольку ^~ 1 ^ 1, S2 < | Е (* "О2 < S fe - tfi)2- |2 тЕг S Заметим, что если xlt хъ ..., д:п>0, то + а:п)2 = д:? + л;1+ ... + х2п + 2 (Xlx2 + д:„_1л:п)~^х\-\- х\Л- ... + *п. Следовательно, S2 < 2 (* -ai)2 < (2(<к - а3+ ... +fln—(л— откуда S<(a! + a2+ ... +an)— n^ или па1^(а1 + а2+ ... +art)—S. (1) Аналогично 3* 67
^Т 2 ( 2 (On —a,)2) ="2 J1^i-{an — t n-1 n—1 < 2 (а„ — а,)2 < (2 (а„ - а,))2 = [па„ — откуда ( S (2) Неравенство (a! + a2+ ... +an)—S^(a1 + a2+ ... +a^)+5 (3) очевидно. Объединив неравенства (1), (2), (3), получим: £(a1 + a2+ ... + ап) — S < (аг + a2 + ... + Найдем условия, при которых имеют место равенства. Легко заметить, что для этого необходимо, чтобы (at■— а1)2 = (а{— ajf при /=1, 2, ..., /— 1 (см. самое первое неравенство). Но тогда ах = а2= ... = а,_ь Так как / = 2, 3, ..., /1, то равенство возможно лишь при ах = а2 = ... = ап = а. Действительно, в этом случае 52 = ^ 2 (a,-a,f = 0 п l \<i<i<n т. е. равенства па1 = (а1 + а2+ ... + а„) — S = (аг + а2 + 5 = пап действительно имеют место. 68
50. Из неравенства -rj- < . . _ „ при k > 1 и тождества т^—.Г) = JLJ-±. получаем: -2-4<2. 5t. Зафиксируем /. Тогда, используя неравенство, доказанное в предыдущей задаче, получим: L у Просуммируем полученное неравенство по / и вновь применим неравенство, доказанное в задаче 50: М N , М _ Й „Й "^F"< 1Й ^2 < 52. Из неравенств min^, -^-J <-^2-, minf-^, ^j< -у имеем: Отсюда, используя неравенство, доказанное в предыдущей задаче, получим: М N , N M N М{^ )
53. Раскрыв скобки, получим Ах2 + Вх + С = а1а2х2+ + (я А+ аА) х + btb2i отсюда | А |< й^ (| ax |< hl9 \ сц |< <й2), |В|<2/1^, |С|<ЛЛ. И следовательно, Я<2/11Л2. Пусть для определенности Лх = | аг |, /i2 = I b21. Тогда, если |61|>-^-1 то |С| = |&А|>-^-. Если \аъ\> >^, то |Л| = Ка2|>-^-. Если I^K^-, то \B\ \a _Jhh1_>Juhz_t Пусть теперь hi==lail9 h2 = \a2\. Тогда Я>|Л| 54. Воспользуемся индукцией по п. а) п = 0. Тогда Р (х) = а0. Пусть | 1 — а01< 1 и |а —ао|< 1. Тогда \а— 1| = |а — ао+ао — 1 |<|а—ао! + + \а0 — 11 < 2, но а^З. Получили противоречие. б) Пусть наше утверждение верно при k^n—1. Рассмотрим многочлен _ Р(х+\) — Р(х) Легко заметить, что Q (х) — многочлен степени п — 1. По предположению индукции найдется такое i (0 — 1), что а—\ 1^+1 — p(t + 1) — а! + Я (01 / | аж — Р (t а-\ а—1 По крайней мере одно из двух слагаемых в окончательном выражении не меньше -=-. Если это первое сла- 70
гаемое, то, так как -^— ^ 1 при а ^ 3, | а'+1 — Аналогично, если второе слагаемое не меньше -^-, то |а'—Р(01> 1. 55. Имеем 3! = 6, (3!)! = 6! = 720, ((31)!)! = 720!. Так как 720! > 100 • 101 . ... . 720 > 100621> 101242, то в разложении ((3!)!)! не менее 1243 цифр. Среди 720 чисел 1, 2, ..., 720 на 5 делятся 144 числа, на 25—28 чисел, на 125 — 5 и на 625 — одно. Поэтому в разложении числа 720! на множители имеется 178 пятерок. На столько ж'е нулей и будет оканчиваться 720!, потому что двоек 'в разложении 720! на множители, как легко посчитать, больше 178. 56. Из того, что 24= 19 + 5, 14= 19 — 5, получаем 24*977 + 141977 = (19 + 5)1977 + (19 — 5)1977 = 19 . А + 51977+ + 19 • В — 51977 = 19 (А + В), где Л и В — некоторые натуральные числа. 57. Заметим, что 1978 —число вида 4& + 2, а 1979 — число вида 4& + 3, где k—натуральное число (в данном случае 494). Пусть D = Ъ2 — \ас = 4& + 2, тогда Ь2 делится на 2 и Ь делится на 2. Пусть Ь = 2/, тогда Ь2 = 4/2 и 4/2 — Аас = 4k + 2. Число, стоящее в левой части равенства, делится на 4, а число, стоящее в правой части равенства, не делится на 4. Получили противоречие. Пусть Ъ2—4ас = 4& + 3, тогда Ь2 нечетное и, значит, Ъ нечетное. Если Ь = 2т + 1, то D = 4т2 + 4т + 1 — 4ас = 4 (т2 + т — ас) + 1 = 4fe + 3. Снова получили противоречие. 58. Дискриминант не может принимать значения 2 и 3 (см. задачу 57). Но 0, 1 и 4 квадраты целых. Так как 7J
5 = 32 — 4 • 1 • 1, то 5 —дискриминант квадратного трехчлена х2 — Зх+ 1. Искомое число 5. 59. Обозначим [f п] через t. По определению /<|/1Г</+1 или /2</г<(/+I)2. По условию п = Ы, где k — натуральное число. Следовательно, Если t = 1, то, как легко заметить, [Y п ] = 1 лишь при /1=1, 2, 3. Пусть теперь /> 1, тогда — < 1, и мы видим, что 3. Так как / и k — натуральные числа, то при фиксированном / k может ^тринимать лишь значения t, t + 1, / + 2, т. е., если [Y п ] =/, то п равно либо /2, либо t (t + 1)» либо t (t + 2). Заметим, что при / = W2 = 1, t (t ■+ 1) = 2, t (t + 2) = 3. Следовательно, все п, удовлетворяющие условию задачи, описываются формулами: n=/2, n=t(tJr 1), л = М' + 2), / € Z. 60. Ясно; что /2=1 и /г = 2 удовлетворяют условию задачи. Если /г^З, то число делителей п по крайней мере 2{\У п ] + 1, [У~аГ] + 2). Первый из них не менее Y п , а второй не менее V* /г + 1. Так как эти делители последовательные натуральные числа, то они взаимно просты. Поэтому Y п (V п + 1)^я, что невозможно. Следовательно, только п = 1 л п = 2 удовлетворяют условию задачи. 61. Пусть /г>625. Обозначим [/г4] через а. Тогда п делится на а, а—1, а—2, а—3, а — 4. Из условия задачи следует, что п не меньше, чем наименьшее общее 72
кратное этих чисел, которое обозначим через d. Числа а, а — 1, а—2, а—3, а — 4 могут иметь в качестве общих делителей лишь числа 2, 3 и 4, поэтому d> 2-3-4 а(а— 1)(а— 2)(а— 3)(fl—4)- Имеем неравенство ^ а (а— \)(а — 2) (а — 3) (а — 4) (а — 4)6 ^ ^ 24 -^ 24 ^ (п^" _ 5)5 s^ 24 Если п>520, то п4 >5/г5. Для таких п _i _L JL 24 ^ 24 ^ 24 ^ ^" Получили противоречие. Значит, п<520. 62. Предположим, что при каких-го п и k числитель и знаменатель этой дроби делятся на d. Так как ап+3 — — ап+2 = ап+\, an+i—ап = ап-\ и разность знаменателя и числителя делится на d, то и (а/г+3£ + ап+\) — (ап+Ж + ап) = an+lk + ап^ делится на d. Аналогично {an+2k + ап)—{an+\k -\~ ап_{) = = ап k -f an-2 делится на d. Продолжив эти разности, получим, что аък + а1 и ajz + а2 делятся на d. Но а3=2, ах = 3, откуда 2k + 1 и 3/г + 1 делятся на d. Их разность (3k +1) — (2k + 1) = k тоже делится на d. Но k и 2fe + 1 одновременно не могут делиться на d> 1. Получили противоречие. 63. Положим d равным минимуму разностей х—у, где х, у£Р и х>у. Пусть d = a — b, где {а; 6}с=Р, покажем, что а делится на d. Представим а в виде а = sd + q, где 0 ^q < d. Тогда * = (s+ \)a£P и t/ = = s (a + d) = s6 £ Р. Поскольку * — у = ^ < d, то в силу 73
определения d x = у, откуда q = 0. Итак, а = sd. Положим К = s2, покажем, что для любого k^K kd^P. Представив k^K = s2 в виде k = st -f г, где 0 ^г < $, имеем / ^ 5. Тогда & = (t — r)s + r (s + 1), где £ — г > 0. Поэтому Ы = (/ — г) sd + г {sd + d) = (t — г) а + rb g P. Покажем, наконец, что любое число х £ Р делится на d. Пусть x = pd + А, где 0 < А < d. Тогда у = (s2d+ \)x £ Р. Имеем y = (s2dp + hs2)d + h. Так как s2dp + hs2^s2=K, то 2 = (s2dp + As2) d g Р. Но у —z = /i < d\ следовательно, в силу определения d у = z, т. е. h = 0 и х = pdt что и требовалось доказать. 64. Пусть задано натуральное число п. Рассмотрим п + 1 число: mt=l, m2=ll, /л3=111, ..., mn+i = = 11 ... 1. Так как различных остатков при делении на п существует лишь п, то некоторые два числа т, и т\ из этих чисел имеют один и тот же остаток. Если i < /, то число т\—т, делится на п и имеет вид 11 ...J^ 000 ...0. 65. Прежде всего заметим, что если т=10&+л, то по формуле Ньютона тт = (10* + n)100 = 1010V00 + 10"?^" + п1ОО= ЮООЛ+n100, так как 1О2С?Оо = 5000 . 99, 10 . Cioo = 1000. Следовательно, последние три цифры числа т100 те же, что и /г1000, где п — последняя цифра числа т. Рассмотрим отдельно четыре случая: а) п = 0. Очевидно, последние три цифры тт—это 000. б) п = 5. Тогда п2 = 25, п* = 625, л4 = 3125, пь = = ...625, /гв= ... 125, ... . Так как 100 число четное, то последние три цифры числа т100 — 625. 74
в) /1=1, 3, 7, 9. Тогда л4 =1,81, 2401, 6561 и я4 = 40 Л + 1, где Л —натуральное. Тогда я100 = (40Л + I)25 = 4025Л25 + Cit • 4024Л24 + ... +С2540Л+ 1 = 1000В+ 1. Следовательно, в этом случае последние три цифры числа /л100— это 001. г) п = 2, 4, 6, 8. Очевидно тогда, что п100 делится на 2100 и тем более /г100 делится на 8. С другой стороны, легко заметить, что п4 при делении на 5 дает в остатке 1, т. е. д4 = 5Л + 1. Тогда т 1)25= 125С+ 1, т. е. п100 при делении на 125 дает в остатке 1. Пусть последние три цифры числа /г100 равны а, Ь, с, т. е. п100 = 1000/С + 100а + 106 + с. Тогда, поскольку 1000 делится и на 8, и на 125, то трехзначное число аЪс делится на 8 и дает при делении на 125 в остатке 1. Но из чисел 1, 126, 251, 376, 501, 626, 751, 876 (это все трехзначные числа, дающие при делении на 125 в остатке 1) лишь одно число 376 делится на 8. Значит, в этом случае последние три цифры числа т100—это 376. Следовательно, три последние цифры числа /л100 — это 000, если последняя цифра т равна 0; 625, если последняя цифра т равна 5; 001, если последняя цифра т равна 1, 3, 7 или 9; 376, если последняя цифра т равна 2, 4, 6 или 8. 66. Пусть р—произвольное простое число, большее п. Пусть Af — такое натуральное число, что N^>p + + (п — \)п\. Положим а0 = N\ + р, а± = N\ + р + п\, a2 = Nl + p + 2 (л!), ..., £*„_! = М + р + (п — 1) (я!). Очевидно, а0, av ..., an_i—арифметическая прогрессия с начальным членом N1 +р и разностью п\. Докажем, что все 75
числа а0) av ..., ап — составные. Действительно, так как при 0^k<n p + k(n\)^N, N1=1-2- ... • N, то число p + k(n\) встречается среди чисел 1, 2, ..., N, т. е. N1 делится на p + k(n\). Следовательно, ak = № + р + k (n\) = (p + k (л!)) с, где с — целое число. Очевидно, с>1 и, следовательно, uk — число составное. Теперь докажем, что при 0 <! & < m <1 я числа ak и ат взаимно просты. Предположим обратное, тогда существует простое число #> 1» делящее ak и ат. Тогда q делит и am—ak, но am—ak = [N\+p + m(n\)]— [Nl + р + k(n\)] = т. e. q делит (m — k) (n\). Поскольку l^m—k<^n, то (m— k)(nl)= 1 • 2 • ... • (m—k — l)(m—k)2 ... п. Поскольку ^ простое, то оно делит по крайней мере одно из чисел 1, 2, ..., (m— k — 1), (т — &)2, (т—£+1), ..., п. Но если q делит (т—k)2, то q делит и т—k. Следовательно, q делит 1 • 2 • ... • (т —k—1) «(/л—&)•... х X п = п\. Поскольку N>n и п\ делит М, то q делит и N\. Но q делит am = N\ + Р + /и (л!). Следовательно, 9 делит и р. Но так как р простое, то q = р. С другой стороны, р > п, а мы видели, что # делит одно из чисел 1, 2, ..., я, т. е. что q^n. Получили противоречие, которое и показывает, что числа а* и ат взаимно просты. Таким образом, мы построили арифметическую прогрессию а0, av ..., ап_ь состоящую из составных чисел, все члены которой попарно взаимно просты. 67. Возьмем в качестве av a2f ..., ал_3 произвольные четные числа, в качестве а„_2 — произвольное нечетное число. Тогда сумма квадратов этих чисел—нечетное число. Пусть а\+а\+ ... +a5_2 = 2fe+l; положим а„_1 = k, ап = k + 1. Тогда а? + а2 + ... + a«-i= 76
+ 1 + k2 = (k + I)2 = al, т. е. построенная последовательность (av a2, ..., an) удовлетворяет условию задачи. Ясно, что таким образом можно построить бесконечное число различных последовательностей. Так как наибольший общий делитель чисел ап-.\ = k и ап = k + 1 равен 1, то построенные последовательности попарно не пропорциональны. 68. Доказательство будем проводить с помощью метода математической индукции. При п = О наше утверждение очевидно: число 2 встречается в 0 (т. е. ни в одной) пифагоровых троек. Пусть наше утверждение верно для любого k^n. Тогда, е:ли (х, у у z)—пифагорова тройка, в которую входит число 2П+2, и числа х, у, z не взаимно просты, то числа имеют общий множитель 2 и f-|-, -у, -|-)—пифагорова тройка, в которую входит число 2n+1. По предположению индукции таких троек ровно п. Пусть теперь (*, у, z)— пифагорова тройка, числа х, у, z взаимно просты, и одно из них равно 2"+2. По условию х2 + у2 = z2. Поэтому, если z четно, то х и у нечетны. И следовательно, х2 и у2 при делении на 4 дают в остатке 1. Тогда х2 + у2 при делении на 4 дает в остатке 2, в то время как 22 делится на 4 без остатка. Получили противоречие. Следовательно, z нечетно и, в частности, z=£2n+2. Но х2 = (z — y)(z + у), и если х = 2"+2, то х2 = 22"+4 = = (z—y)(z + y)> откуда 2 — y = 2k, z + у = 22"+4~* (О < k < 2n + 4). Следовательно, z = \ (2k+22n+4~k). Так как z нечетно, то либо k = 1, либо k = 2n + 3. В перЕом случае 2=1+ 22"+2, у = 22"+2 — 1, х = 2"+2 и, так как 0 < 2"+2 < 22"+2 — 1 < 22"+2 +1 при п > 1, то (2"+2, 22"+2 — 1, 22/1+2+ 1) —пифагорова тройка. Во втором случае 2=1 + 22"+2, у = 1 — 22"+2. Так как в последнем случае у < 0, то существует ровно одна пи- 77
фагорова тройка (jc, у, z), такая, что х, у, z взаимно просты и х = 2П+2. Пусть, наконец, у = 2п+2, тогда, рассуждая, как в предыдущем случае, получим, что г = 2"+2 + 1 и либо х = 22"+2 — 1, либо х = 1 — 22"+2. Но при п > 1 в первом случае *>#, а во втором—*<0. Таким образом, существует ровно одна пифагорова тройка (л:, у, г), такая, что х, у, z взаимно просты и одно из них равно 2"+2. Поэтому всего пифагоровых троек, в которые входит число 2"+2, ровно п + 1, что и требовалось доказать. 69. Обозначим сумму всех делителей числа п через D(n). Нам надо показать, что D (п) < п |/ п при Разберем сначала случай п = 2а(а—целое число, а Тогда D (п) = 1 + 2 + 22 + ... + 2а = 2а-Н _ = п VT. Предположим теперь, что пф2а и D (k) <& V k для всех 3 ^ А < я, и покажем, что тогда D (п) < п V п. В силу вышесказанного можем считать, что п = шру где р — нечетное простое. Заметим, что при р ^ 3 1 + Р < Р V Р (действительно, при р ^ 4 1 4 <2< <У"~р~; при р = 3 1 +_3_< 3 V 3 ). Поэтому, если /и = 1, то D(p) = 1 + р < р У" р ; аналогично, если m = 2, то D (л) = 1 + 2 + Р + 2р = 3 + Зр < 2р У"2р = /г У^ если же m^3, то по предположению индукции D(пг) <^пгУ т. Так как любой делитель числа п имеет вид d или dp, где d — делитель /л, то D(n) = D (т) + pD (m) = D (m) (1 + р)< 78
70. Обозначим через S множество всех пар натуральных чисел (х7 d), таких, что d делит х и x^.d2^n2. Ясно, что S содержит в точности Nn (1) + Nn (2) + ... + + Nn (n2) элементов. Но (х, d) £ S тогда и только тогда, когда х = qdy где l^q^d, т. е. S состоит из множества пар (qdt d), где l^<7^d<^. Следовательно, в S столько же элементов, сколько существует пар (q, d)t таких, что l^^^d^ft- При фиксированном d таких пар d. Сле- довательнб, всего таких пар 1 + 2+ ... + п= n ^ ч т. е. Nn(l) + Nn(2)+ ... + Nn(n2) = n(n + l\ 71. Обозначим сумму, о которой идет речь в условии задачи, через f(n). Легко заметить, что те слагаемые, которые входят в f{ri), но не входят в f (п — 1), имеют вид ар = —, где 1<Ср<я и р взаимно просто с п\ те рп слагаемые, которые входят в f (п — 1), но не входят в / (п), имеют вид Ьр = _ ,, где 1 < р < п — р (или 0 < р < -^-) и р взаимно просто с я (или, что то же самое, р взаимно просто с п—р). Таким образом, разность f(n)—f(n — 1) —это сумма чисел вида ар + ап-р— — Ьр у где р пробегает множество натуральных чисел, меньших -£- и взаимно простых с р. Но ~рп •" (/г — ап-р—Ьр = — + ,„^„ — п/„,л = 0. Следовательно, f(n) = f(n-l)= ... =/(2) = -1- 72. Из формулы Ньютона следует, что 79
Xk ± yiV.k & + 1)_пРи четном п, ukVk+\±VkVk при нечетном п, где Xk, Ук, «л, Vk — натуральные числа. Другими словами, при любом п Т + УТ)п = УЖ + VW, (VT+1 - YT)n = УЖ - VЖ, где Nk , Mk — натуральные числа. Перемножив эти равенства, получим 1 = Nk — Mk , откуда следует, что Mk = Nk — 1 при k = 1, 2, ... . Следовательно, —1 — У m—2 i/T -т/т = Сложив эти равенства, получим: что и требовалось доказать. 73. Прежде всего заметим, что если р простое, то f(P) = P+ 1» т- е-не верно, что всегда f(n)<^n. Однако верно следующее утверждение: если п—составное число и п^9, то f(n)^n—2. Докажем это утверждение индукцией по п. Если я = 9, то f(n) = 1 -f 2«3 = 7^9—2. Пусть утверждение верно для любого такого составного k, что 9 < k ^ п — 1. Пусть л = fe^, где ^>1 и £2> I. 80
Если kx простое, то f(kx) = kx + 1. Если же kx составное и ^х^Э, то по предположению индукции f(fti)<^fti—2. Кроме того, f (4) = 5, / (6) = 7, / (8) = 7. Таким образом, для любого ftx<rt f(k1)^.k1+ I (4, 6 и 8—единственные составные числа, меньшие 9). Аналогично f(k)^kl Заметим, что Действительно, если кг = ра{1 ... р°^ и ft2= рь^ ... ръ™ — разложения kx и k2 на простые множители (а/^0, 6/^0), то 1 + alPl + ... + amPmt f(k2) = 1 + blPl + ... + + ЬтРт* 2) = 1 + (ax + bjpx + ... + (am + bm)pm = Следовательно, / W = f (kx) + f (k2) - 1 < (k, + 1) + (k2 + 1) - 1 = = *i + К + 1 = (n + 2) -£Л + kx + k2 - 1 = = (n + 2)-{k1-\){k2-\). Но так как kx > 1, Л2 > 1 и п = kxk2 ^ 9, то (kx — 1) x X (k2 — 1) > 4 (действительно, если (Ах — 1) (k2 — 1) < 4, то либо Лх— 1<1, ft, —1<3, либо kx — 1<3, fta—1<1. В любом случае ^2^8). Следовательно, / (n) = п + 2 — (кг — 1) (ft, — 1) < п + 2 — 4 = п — 2. Наше утверждение доказано. Теперь уже нетрудно получить решение задачи. Действительно, если я^9 составное, то f(n)^n—2. Если же п простое, то f(n)=n+l составное и f(f(n)) ^п—1. Таким образом, за один или за два шага мы всегда можем уменьшить п по крайней мере на 1. Следовательно, в конце концов получим число, меньшее 9. Ясно также, 4 Зак. 2126 81
что для любого п f(n) > 2 (если f(n) = 2 = 1 -f- 1, то 1 = ар, где р—простое, чего не может быть). Если /(п) = 3 = 1 + 1 . 2, то п = 2. Если f (/г) = 4 = 1 + 1 . 3, то п = 3. Если / (/г) = 5 = 1 + 2 • 2, то п = 22 = 4. Если f (n) = 6 = 1 + 1 • 5 = 1 + 1 . 2 + 1 • 3, то либо п = 5, либо п = 6. Следовательно, если /(/г)<7, то и я < 7. Из этого следует, что если дг^9, то в последовательности п, /(/г), /(/(/г)), ... не может встретиться число, меньшее 7. Следовательно, в конце получим одно из чисел: 7, 8, 9. Но /(7) = 8, f(9) = 7, /(/(9)) = 8, и в последовательности /г, /(/г), f(f(n)), ... обязательно встретится число 8. 74. Обозначим через dx наименьшее положительное число среди чисел вида am + bn, где тип независимо друг от друга принимают целые значения. Любое число вида am + bn будет в этом случае делиться на йх = ато+ + Ьп0 без остатка, так как в противном случае мы выполним деление с остатком и получим: ат+ bn = t(am0 + bnQ) + г. Если число г ф 0, то оно, будучи меньше dl9 имеет вид а (т — tmQ) + b(n — tnQ), что противоречит выбору d1# Значит, dx делит а-1 + 6-0 = а и а-0 + 6-1 = 6, т. е. dx — общий делитель а и 6, а поэтому делит d. Отсюда получаем dx^d. С другой стороны, d делит а и 6, а значит, и любую комбинацию вида am + bn, т. е. d делит d1# Получаем d^.dv Неравенства d^dx и dt^.d возможны только при d = dt. а 75. Пусть 6 > а. Возьмем натуральное & > ь__а- Тогда число /г* = (b — a)k + 1 — а будет натуральным. Числа a + nk = (b — a)k+ 1 и 6 + nk = (6 — a)(&+ 1)+ + 1 взаимно просты, так как их общий делитель должен делить их разность Ь — а, и, значит, а + /г/г — (b — a)k= = 1. Придавая k значения kOi ko+ I, ..., где k0 — любое 82
натуральное число, большее ~j~~a> получим бесконечно много таких пи . 76. Пусть х — какое-либо натуральное число. Если ах + с и Ь взаимно просты, то х — это искомое d. Если Ръ Ръ ...» Ps — все те простые числа, которые делят наибольший общий делитель чисел ах + с и 6, а Ps +р -.., Рг —оставшиеся простые делители числа 6, то числа а (х + ps+i Ps+2.. .рг) + с и Ъ взаимно просты, т. е. в качестве d можно взять число х + ps+i ps+2 ... Pr . В случае г = s можно взять d = х + 1. 77. Обозначим через rv r2 гп остатки от деления чисел — av —а2, ..., —ап на т. Так как т>п, то существует такое г(0^г^т—1), что г Ф ri ни при каком / = 1,2, ..., /г. Среди чисел ах + г, а2 + г, ..., а„ + г ни одно не делится на /п. В самом деле, пусть т делит at + г при некотором i = 1, 2, ..., /г. Тогда из представления а/ + г =—(—щ ) + г = — (тб/ + г/ ) + г =—mbi + r— rt получаем, что т делит г — г/, чего быть не может, так как \r — rt |<m и гфгь . 78. Возьмем N > max (2k, 2n), т ^ N. Из равенства 1 заключаем, что все at ^N n . Поэтому число всех которые можно представить в виде суммы (*), не превос- ходит (N) п . Если предположить, что все т^> 2 представлены в виде суммы (*), то получим неравенство ~. Но ~ Получили противоречие. Поэтому существует т>—- > >/С, которое нельзя представить в виде суммы (*). 4* 83
79. Будем предполагать сначала, что К > 4. Возьмем у = р, где р — простое число, р > К. Можно найти такое х9 что х2<.у3<(х + I)2 (у3 должно располагаться между квадратами последовательных чисел). Возьмем разность между у3 и ближайшим к нему числом х2 или (х + I)2. Эта разность по модулю не больше половины расстояния от х2 до (х + 1)2> т- е- v ; = х -\ , а так как эта разность — целое число, то она не превосходит х. Из неравенства х + 1 > з з > у 2 > К 2 видим, что при & > 4 я >/С. Если К ^4, то х и у у найденные выше, больше 4 ^ К, что и требовалось доказать. 80. При п > 4 из равенства -^+i> - 1 = <«=Ш получаем, что число п 1 составное. В самом деле, при n = 2k имеем п(п + Х) —l = (2k—\)(k+ I), a при л = 2k + 1 имеем п(п + 1) 1 = *(2* + 3). Если я = 2, то получаем первое простое число 2, если п = 3, то получаем второе простое число 5. 81. Пусть пф2т. Тогда его можно представить в виде п = //, где / — некоторое нечетное число, большее 1. Отсюда получаем, что число 2п+ 1 =2" + 1 =(2< + l)(2*<'-i> — _2<('-2)+ .. _2' +1) составное. 82. Возьмем п = 3k + 2, где Л — натуральное число. Из равенства /г2 = х2 + р следует, что р = п2 — х2 = (/г — — х)Сп + х). Так как р простое, a n>jc, то я —х=» 1, а п + х = р. Отсюда 84
р = 2/г — 1 = 3(2£+1), что невозможно. Значит, числа вида 3k + 2 (их бесконечно много) нельзя представить в виде х* + р. 83. Для некоторого натурального t обозначим через Mt множество натуральных чисел вида l)! + 2, t(K+1)1 + 3, ..., t(Kl) (Kl)} Множества Mlt M2, ..., Мл , ... — искомые. 84. Так как р > 5, то —— > 2. Поэтому (р — I)2 = = 2- ——(р—1) делит (р—1)!. Пусть для некоторого натурального т имеет место равенство тогда (р— I)2 делит рт — 1, откуда р— 1 делит + рт-2 + л л л + р + j = (pm-i _ 1) + Поскольку р— 1 делит р1 — 1 при любом / = 1, 2, ..., т— 1, то отсюда следует, что р— 1 делит т. Поэтому т>р— 1 и рт>рр~1>(р— 1)р~{+ 1>(р—1)! + 1, что противоречит условию. 85. Число р4 имеет пять натуральных делителей: 1, р, р2, р3, р4. Пусть 1 + р + Ра + Р3 + Р4 = /г2. Из этого равенства можно получить: (2/г)2 = 4/г2 = 4 + 4р + 4р2 + 4р3 + 4р4 > > 4р4 + 4р3 + р2 = (2р2 + р)2, (2п)2 = 4 + 4р + 4Р2 + 4р3 + V < < 4р4 + р2 + 4 + 4р3 + 8р2 + 4р = (2р2 + р + 2)2, т. е. 85
(2р2 + pf < {2nf < (2p2 + p + 2)2, откуда (2az)2 = (2p2 + P+ l)2. Поэтому 4/г2 = 4p4 + 4p3 + + 5p2 + 2p + 1. Но, с другой стороны, 4д2 = 4(р4 + р3 + -j-p2+p+ 1). Вычтем из первого представления для 4/г2 второе. Получим р2 — 2р — 3 = 0, р = 3. 86. Для определенности будем считать, что разность прогрессии положительна. Докажем, что d делится на 2, 3, 5, 7, 11, 13. Отсюда будет следовать, что d делится на 2.3.5.7-1ЫЗ = 30 030, а поэтому d>30000. Обозначим члены нашей прогрессии через av а2, ..., а1Ь. Пусть 9=2, 3, 5 или 7. Если для некоторого i имеем at = qy то либо i—1><7, либо 15 — i^q. Это вытекает из того, что 15>2<7+ 1. Предположим, что имеет место второй случай: 15 —1^><7- Рассмотрим остатки при делении на q чисел а,+ь а^, ..., а*+?. Так как эти числа отличны от q, то возможны остатки 1, 2, ..., q—1. Из того, что чисел больше, чем остатков, вытекает существование двух чисел cii+k, CLi+i, где 1^^</^?, с одним и тем же остатком. Тогда число ai+i — а/+л = (I — k)d делится на q, а так как 1^/ — k^q—1, то d делится на q. Если же ни одно из чисел щ не равно qt то мы можем повторить те же рассуждения для чисел аъ аъ ..., aq и получим, что d делится на q. Таким образом, d делится на 2, 3, 5, 7, следовательно, d>2-3-5.7 = 210. Пусть теперь q= 11 или 13. Так как q < d, то ни одно из чисел а2, Од, .. .,а1Ь не равно q. Рассмотрев q чисел а,, а^у . . ., aq+\ точно так же, как и выше, получим, что d делится на q. 87. Для р = 2 можно взять, например, х= 1, у = 2. Пусть р>2. Покажем, что если 0^а <&^р"~ , то а2 и б2 дают при делении на р различные остатки. В самом деле, если а2 = pq + г, б2 = р/ + г, где 0 ^ г < р, то б2 — а2 делится на р. Но Ь2 — а2 = (Ь + а) (Ь — а), где b + а<р и Ъ — а <р. Отсюда следует, что различные 86
остатки при делении на р дают также число вида —а2— — 1, где 0^а<; р~ . Рассмотрим р + 1 число -1 -2'-1.-1'-1.-1А () () I2, 22, ..., |р~"~ ) . Так как при делении на р возможно лишь р остатков, то по крайней мере два числа из этих р + 1 чисел будут иметь равные остатки. По доказанному выше, если одно из этих чисел имеет вид х2, где 0^х<1р~ , то второе обязано быть вида—у2—1, где 0<j/<p~ . Тогда х2 + у2 + 1 делится на р. 88. Сначала докажем, что если п простое, то k > — и 4 I> —. Пусть kn + 1 = а2, тогда kn = (a— 1) (а + 1), 4 т. е. п делит либо а— 1, либо а + 1. Если а + 1 = пЬ9 то а — 1 = nb — 2, а если а — 1 = nb, то а + 1 = nb + 2. В любом случае kn > п& (пб — 2) > п (м — 2). Следовательно, £>д — 2. Но прил>3 /г — 2 >— ,т.е. £>—. 4 4 Далее, поскольку //г — точный квадрат и /г простое, то / = п >—. 4 Пусть теперь &>— и />—. Докажем, что в этом 4 4 случае п простое. Предположим противное, т. е. что п— составное число. Рассмотрим два случая; а) п — степень простого числа, т. е. п = ps, где р — простое и s > 1. Очевидно, если 5 четно, то п = 1 • п — точный квадрат, I = 1 < —. Если же s нечетно, то рп= 4 87
— точный квадрат и / = р = ps = —^-. Так ps—1 ps—1 как s>3 и р>2, то ps~l > 4, откуда /<—. Следоваг тельно, в этом случае /<—. Получили противоречие. б) /г делится по крайней мере на два различных простых числа. Тогда п можно представить в виде п = st, где s и t—взаимно простые числа, большие 1. Докажем, что тогда можно выбрать такое натуральное число ху что 1^#^я — 1, s делит х— 1, t делит х+1. Для этого рассмотрим числа t — 1, 2/ — 1, ..., st—1. Таких чисел s. Если хотя бы два из них, например at — 1 и Ы — 1, где 1 ^ а < b ^ s, дают одинаковый остаток при делении на s, то число (bt—\)—(at-l) = (b—a)t делится на s. Так как s взаимно просто с /, то Ь—а делится на st но 1<1&—a<s. Получили противоречие. Следовательно, все остатки от деления на s чисел / — 1, 2t—1, ..., st—1 различны. Но различных остатков от деления на S ровно s, это числа 0, 1, ..., s— 1. Следовательно, для некоторого a(l^a^s) число x = at — 1 при делении на s дает в остатке 1 или, что то же самое, х— 1 делится на s. С другой стороны, очевидно, что х + 1 делится на /. Поскольку 1<х<^ — 1 = п — 1, то число х — искомое. Пусть теперь а — меньшее из чисел х и /2 — х. Поскольку а^—, то а9 ^ —. Так как х2— 1 = (х— 1)(х+ 1), (п—х)2 — 1 = {п—х— \)(п — jc+ 1), числа х — 1 и п — х+ 1 делятся на s, а числа х + 1 и п — х— — 1 делятся на t, то а2 — 1 делится на п, т. е. а2 = = krn + 1. Следовательно, k^kx = —— < — ^ —, т. е. п п 4 к < —. Получили противоречие. 88
Следовательно, п — простое число. 89. Пусть 2 = рг < р2 < р3 < ... — последовательность простых чисел. Так как при п> 1 рп —число нечетное, то нетрудно доказать, что при п>7 рп > 2п + 1. Обозначим через уп сумму Заметим, что у7 =2+3 +5 +7+ 11 + 13 + 17 = 58 > 49 = = 72. Докажем индукцией по /г, что при /г>7 #Л>/г2. Действительно, уА+1 = у* + рЛ+1 > £2 + (2й + 1) = (Л +1)2 при k > 7. Кроме того, */х = 2 > I2, у2 = 5 >22, t/3 = 10> > З2, у, = 17 > 42, f/5 = 28 > 52, уб = 41 >62, т. е. мы доказали, что при любом п Уп^-п2- Пусть т2—наибольший из квадратов, не превосходящих" уп. Тогда по выше- доказанному т > /г, т. е. т = п + kt k > 0. Таким образом, мы доказали, что для любого п > 1 существует такое k ^> 0, что Докажем, что тогда pn+i > 2 (я + k)-\- 1. Предположим противное, т. е. что pn+i^.2(n + k) + 1. Но при п>2 Рл+i > Рп + 2. Следовательно, Р«<2(л + А)+ 1—2 = 2(п + Л) —1, + £)+ 1 —4 = 2(n + ft) —3, 3 = />2 < 2 (л + k) + 1 — 2 (п — 1) = 2 (п + k) — (2« — 3), 2 = Складывая все эти неравенства, получим: Уп = р! + р2+ ... + />n<2rt(n + fc)-(l+3+ ... + (2п— 1)) = 2п (п + k) — п2 = па + 2nk + 1 — 1 = = (л + *)2 - 1, 89
т. е. уп ^ (л + ky — 1. Но по предположению уп > >(м + k)2. Получили противоречие. Следовательно, 2(n + k) + 1. Тогда уя + Р« >(л + £)2 + 2 (л + й) + 1 = = (n + k+l)*>ynt т. е. Уп<(п + к+1)*<уп+1. Следовательно, (п + k + I)2 лежит между 0я = Pi + Л + • • • + Рп и уп+1 =Pl + p2+ ... + рп+{. 90. Различных объединений УИ^ U Mit [} . . . (J М^, составленных из множеств Л41э М2, . . ., Мп+\, столько же, сколько непустых подмножеств \ilt i2, . . ., ik } множества {1, 2, . . ., п + 1}. Число их равно 2п+1— 1. Так как число непустых подмножеств множества S равно 2п—1< < 2"+i—1, то среди этих объединений хотя бы два должны давать одно и то же подмножество S: A^U М<21) ... U Mik = Mu[}Mu U ... [}Mfr 91. Положим So = 0, Sk =а1 + а2+ . . , + ak при^= = 1, 2, . . ., /г. Рассмотрим п+ 1 число So, Slf . . ., Sn. В силу принципа Дирихле1 найдутся такие t, / (1 ^t < </^п), что S^ и Sy дают при делении на п одинаковый остаток. Следовательно, S/ —St = а^, t + af., 2 + ...+ + а. делится на /г. 1 Принцип Дирихле заключается в следующем: если задано отображение множества М из п + 1 элемента во множество N из п элементов, то в некоторый элемент множества N перейдут по крайней мере два элемента множества М. В шутливой форме это часто выражают так: если посадить я + 1 зайца в п клеток, то в некоторой клетке окажется не меньше двух зайцев. Подробнее о принципе Дирихле можно прочитать в февральском номере «Кванта» за 1977 год статья В. Болтянского «Шесть зайцев в пяти клетках»), 90
92. Для произвольного натурального числа а через f(a) обозначим наибольшее нечетное число, делящее а. Другими словами, a = 2Pf(a), где f(a)—нечетно. Рассмотрим п+1 число /(aj), f(a2), . . ., /(an+i). Все они по условию не превосходят 2п. Но существует лишь п различных нечетных натуральных чисел, не превосходящих 2/г. В силу принципа Дирихле найдутся такие i, j(l<i<j<n+ 1), что f(a.) = f (ау) = k. Тогда щ = 2р k, dj = 2? &, и, так как, очевидно, q > p, то а/ делится на аг 93. а) Положим S* = ах + я2 + . . . + я* . Рассмотрим числа 0, ах — аП9 S1$ S2, . . ., Srt_i (всего п + 1 число). В силу принципа Дирихле по крайней мере два из них дают при делении на п одинаковый остаток. Возможны четыре случая: 1) аг—ап делится на п. Тогда ап == аг + kn. Заметим, что, поскольку ах + а2 + . . . + ап = 2п и щ > 1, то art ^ 2/г — (п — 1) = п + 1. Кроме того, так как по условию art=^tt+ 1, то ап<п+ 1. Следовательно, % + + kn < я + 1, откуда fe = 0 и аг = ап. Следовательно, аг = а2 = . . . = ап = 2. Если п = 2/л, тоа1 + а2+...+ + От = П. 2) S/ —Si делится на п. Если /> i\ то 1^5/ — 5/ = = аж + a,i+2 + . . . + af < 2n— 1, откуда a/+i + a/+2 + + . . . + a/ = /г. 3)S/ делится на /г. Но 1 < S/ <Srt_i = 2п—ап <2п. Следовательно, 5/ = ах + а2 + . . . + a,i = п. 4) 5/г и %—ап дают одинаковый остаток при делении на п. Тогда, если k= 1, то аг — (аг — ап) = ап делится на п. Но (см. случай а)) 1^ал</г+ 1, откуда ап = п. Пусть теперь &> 1, тогда Sk —(a1—an) = a2 + a3+. . .+ + ak + ап делится на п, 1 < а2 + а3 + .. . + ak + ап < < 2/г — ах < 2/г, откуда а2 + а3 + . . . + ak + ап = п. б) Решается аналогично. 94. Пусть задана дуга РгР2. Уменьшив, если необхо- 91
^—' 2л димо, эту дугу, мы можем считать, что РХР2 = —, где /72 т — натуральное число. Построим точки P3t P4, ...» Рт так, чтобы точки Plt P2t . . ., Рт делили дугу окружности на т конгруэнтных частей. Возьмем некоторую точку Qo £ А и обозначим через Qn точку, в которую переходит Qo при повороте на п радиан. Покажем сначала, что множество точек A(Q0) = = {Qo» Qi> • • •» Qn, . . .} бесконечно. В самом деле, если оно конечно, то найдутся такие различные s и tt что Qs = Qr Если s < /, то отсюда следует, что точка Qs переходит в себя при повороте на / — s радиан. Так как дуга всей окружности равна 2я, то это означает, что / — s п = , т. е. мы пришли к противоречию с иррациональностью числа я. Итак, A(Q0)—бесконечное множество. Так как множество дуг РХР2, Р2Р3, . . ., РтРх конечно и покрывает всю окружность, то найдется некоторая дуга Рь Pk+{, содержащая по крайней мере две точки Qs и Q, из A(Q0). Таким образом, величина дуги QsQt равна а < —^-. Как т показано выше, Qs Ф Qt , т. е. а Ф 0. Рассмотрим последовательность точек Ro = Qs , Rx=Qt , ..., Rn = Qs+n(^s),.. . из A(Q0). Для любого п дуга RnRn+{ получается из v^/?0/?i = ^Qs Qt поворотом на n(t —s) радиан. Следова- ^ 2л тельно, Rn Rn+i = а. Ясно, что для п > —дуги R0Rly R1R2, . . ., Rn Rn+\ покроют всю окружность. Пусть точка Р1 принадлежит дуге Rk Rk+i. Поскольку /? < F\P2> то либо Rk+u либо Rk-\ принадлежит дуге РХРЪ что и требовалось доказать. 95. Единичным квадратом будем называть квадрат с вершинами в точках (н, v)9 (u+ I, v), (u+ I, v+ 1), 92
(м, v+ 1), где и, у—целые числа. Пусть Kv К2, . . . ♦ /Cm— единичные квадраты, содержащие хотя бы одну точку данной фигуры Ф. Обозначим через Ф< (/=1, 2, , . ., т) пересечение Ф и Кг Через Ф, обозначим образ Ф[ при параллельном переносе плоскости, переводящем квадрат Ki в квадрат К с вершинами (0, 0), (1, 0), (1, 1), (0, 1). Сумма площадей Ф^. (i =1, 2,. . ,,m) равна сумме площадей Ф, и, следовательно, площади Ф. Так как все Ф; лежат в квадрате К площадью 1, а площадь Ф по условию больше 1, то существует точка Л^/С, принадлежащая по крайней мере двум фигурам Ф'г Пусть А £ £ Ф\ [)Ф'2 и является образом точек А1 € Фх и А2 £ Ф2# Тогда, если (а, р) — координаты Л, где 0<а< 1, 0^ <Р< 1, то координаты Ах и А2 имеют вид (иг + а, ух + + Р) и (и,, + а, у2 + р), где их, w2, t;lf v2 —целые числа. (Это вытекает из явной записи определенных выше параллельных переносов.) Ясно, что Av A2 — искомые точки. 96. Рассуждая так, как и при решении предыдущей задачи, убедимся, что существует не менее чем п + 1 точек плоскости, у которых дробные части координат одни и те же (пусть (х% у)). Совершим параллельный перенос всей фигуры так, чтобы одна из точек с координатами (т + х, п + у) (т> п — некоторые целые числа) стала целой. Автоматически целыми станут и все остальные точки, дробные части координат которых (х, у). 97. Так как угловой коэффициент прямой —< Y 3» о то прямая наклонена к оси абсцисс под углом, меньшим 60°. Поэтому \п—у(т)\ меньше удвоенного расстояния точки М до прямой. Имеем: 1 / 5 4 1 [15/1 — 25т—12| 15 ' I 3 5 I 15 93
Но 15/г—25m— 12 — число целое, значит, неравенство возможно лишь тогда, когда 15/г — 25т = 12. Полученное уравнение не имеет решения, так как его левая часть делится на 5, а правая нет. Следовательно, на указанном расстоянии целых точек нет. 98. Возьмем произвольное целое <7>0. Пусть aq попадает в интервал \р\ р + 1], т. е. /?<а^^р+ 1. Если \aq — р\ <! — или |а<7 — (р+ 1)| ^—, то искомые р> о о >0 и ?>0 найдены. Если же ни одно из этих неравенств не выполняется, то а<?£]/Н ; р-\ [.Вэтом 3 о случае \2<xq—(2р+1)|<—, и в качестве искомых це- «з лых можно взять 2<7 и 2р+ 1. 99. а) Доказательство будем вести индукцией по п. Если п = О, то в качестве круга /Со, не содержащего внутри себя точек с целыми координатами, можно взять любой круг с центром в точке О(х0, у0), где хотя бы одно из чисел xOt у0—нецелое, и подходящего радиуса г. Если, например, х0 — нецелое число, то можно в качестве г взять наименьшее из чисел {х0} и 1 — {a:0}. В самом деле, если М(х, у) — внутренняя точка построенного таким образом круга Ко, то х < х0 + \ОМ\ < х0 + г ^ <хо+ 1 — {хо} = [х0] + 1 их>л;о— |ОМ|>х0 — г>*о— — {*о} = [*оЬ т- е- [*о] < х < [хо] + 1, а поэтому л: —нецелое число. Предположим, что мы уже построили круг Кп с центром Оп у содержащий внутри себя ровно п точек с целыми координатами. Будем увеличивать радиуо круга, оставляя центр на месте, до тех пор, пока на окруж- 94
ность не попадет какая-нибудь точка с целыми координатами. Получим круг Llt концентричный кругу Кп, содержащий внутри себя п точек с целыми координатами, лежащих в Кп, и по крайней мере одну такую точку А на окружности. Пусть Р и Q — точки пересечения (ОпА) о окружностью Кп, причем Оп лежит между Р и А. Построим круг L2 с диаметром [РА]. Круг L2 содержит в точности п + 1 точку с целыми координатами: п точек внутри себя и одну точку на границе. Можно теперь, немного увеличив радиус круга L2, получить круг Кп+и концентричный L2, который содержит эти п + 1 точек внутри себя и не содержит других точек с целыми координатами (рис. 6). б) Из анализа решения задачи э) вытекает, что необходимость в переходе от круга Ьг к кругу L2 была вызвана тем, что круг Lx мог содержать на границе более одной точки с целыми координатами. Если бы мы были уверены в том, что это не имеет места, то можно было бы, немного увеличив радиус Lv получить ИСКОМЫЙ Круг /Ог+1» концентричный кругу Кп • Из этого замечания видно, что наша задача будет решена, если мы найдем такую точку О, рис q что любой круг с центром в О имеет на границе не более одной точки с целыми координатами. Другими словами, все расстояния от О до точек с целыми координатами должны быть по- 95
парно различны. Докажем, что точка О с координатами /У 2 , —) обладает этим свойством. Предположим, чтз точки A(av a2) и B(bv 62), гДе fli» ^2» ^i» ^2—целые числа, находятся на одном и том же расстоянии от О. Тогда {аг — V 2)*+ Iа2 — - откуда г — О О О 9 2 О //■» А \ Л/ О — л» I /ч А А | /t» jy \ 11 ^22* В правой части равенства стоит рациональное число. Так как Y~2 иррационально, то аг = Ьх. Следовательно, или (a2—b2)la2 + b2 — -М = Так как a2, fe2—целые числа, то отсюда следует, что а2 = 62. Таким образом, А = В, что и требовалось доказать. 100. Предположим, что при некотором п будет целым числом. Пусть & наибольшее целое с условием 2k ^n, а Г произведение всех нечетных чисел, не превосходящих /г. В этом случае все числа вида 2-1^1 при 2^/г^/г будут целыми за исключением одного, у которого в знаменатель 2 входит в степени k. Поэтому равенство 96
Sn 2k-lT = 2*-T + 2LJT + . . . + 2—I 3 n противоречиво потому, что слева стоит целое число, а справа нецелое. 101. Заметим, что Sn+i =Sn H . Числа S10oohS26o имеют разные целые части, так как s s + ^+ + + J + + > Поэтому для некоторого п (250 < п < 1000) верно неравенство Sn ^ k, Sn+i > k, где k — натуральное число. Очевидно, что 102. Пусть данный многочлен имеет рациональный с— —, ъ . Тогда корень с— —, где а и Ъ—целые взаимно простые числа ъ Чт)ЧтГ + -■■+<■. (т откуда ап + ап-\ап-хЬ + . . . + а^аЬ1-^ + aobn = 0. Полученное равенство противоречиво потому, что ап—\пп—хЪ + . . . + a1abn~{ + uqI/1 делится на 6, а ап не делится. 103. Запишем Р(х) в виде Р(х) = аАх* + а3х3 + а2х2 + ахх + а0. P(0) 7t 97
P(l) — Я (— 1) = 2ах + 2а3=>2(аг + а3) = Р(1) + Р (-1) = 2ао + 2^ + 2а4 = е= 2 (я2 + аи) + 2.7<0 =^а2 + а4 = 7^3, Р(2)—Р (—2) = 4ах + 16а3 = 4 (ах + 4а3) = = 7/4 =*- ах + 4а3 = 7/5» Р (2) + Я (-2) = 2а0 + 8 (а, + 4а4) =*- а1 + ^3 = ^2 I } =^аг и а3 кратны 7; «1 + 4а2 = 7L _ § —т~ ь*2 ** ь*'4 кратны /. з4 = 7гб) Примерами многочленов большей степени, удовлетворяющих условию задачи, могут служить многочлены вида 104. Из разложения многочлена Р(х) на линейные множители Р(х) = (х — аг) (х — а2)...(х— ат) заметим, что все его коэффициенты ат—ь . . ., ^i» ^o выражаются как некоторые функции от корней многочлена где суммирование ведется по всем j\ < /2 < . . . < /*, принимающим независимо друг от друга значения 1, 2,. . ., т. Число слагаемых в сумме не превосходит тт и любое из них не превосходит Nm. Поэтому |am_/| <>nm Nm для любого i= 1,2, . . ., m. Число многочленов Я (л:) с таким ограничением не превосходит (2mmNm + \)m. Это число может быть принято зэ границу для Мт (N) сверху. 105. Пусть некоторое целое t является корнем данного многочлена 98
Р (х) = апхп + ап-{хп~{ + . . . + агх + а0. Представим t в виде t = nk + г, где k целое и ^ п — 1. Тогда Р (г) = Р (г) -Р (0 = (V« + ап-{гп-{ + ...+а1г+а0)- — (ап (nk + rf + ап_х (nk + г)»-' + . . . + а± (nk + г) + + а0) = An, где А—некоторое целое число. Равенство Р(г) = Ап противоречиво, так как в силу условия задачи Р (г) Ф О, \Р(г)\<п. 106. Пусть Р(х) имеет вид Р (х) = ап хп + ап-{х"-1 + . . . + агх + а,. Так как lim Р(х) = оо, то существуют т, при котором Р (т) = а > 1, и t, при котором P(ta + m) > P (m). Но т) = ап (to + "О" ( ) ) ( + О + ( ) *! (to + m) + а0 = Аа + Р (т) = (А + 1) а, где Л> 1 — некоторое натуральное число. Получили, что P(ta-\-m)—составное число. Следовательно, не существует многочлена, удовлетворяющего условию задачи. 107. Предположим противное: пусть существуют простые plt р2, . . ., ps и многочлен Р (х) = ап хп + . . . + +агх + а0, который при всех целых х принимает значения вида р*Мр%*хК . .р£1*\ где аг(х)% а2 (х), .. ., as (x)—неотрицательные целые числа. Далее считаем апф0. Будем придавать х значения bi = (рхр2 . . . ps)1 , /= 1,2, .... Так как с ростом / P(bi) можно сделать больше сколь угодно большого наперед заданного числа, то для некоторого / = /0 min (/0, max (аг (bt)y a2 (bt), . . ., as (bt ))) > т, где т—наибольшая степень, с которой одно из чисел Pv Ptf . . ., Ps входи г в разложение а0 на простые мно- 99
жители. Положим аг (bi ) =av . . ., as (bi) = as. Тогда равенство P ((РЛ . . . Ps )'•) = (рл . . . ps )'• (ал (РЛ . . . P. J^1-1* + a0 = p^p«« . . . р/Ъ противоречиво, потому что некоторое р™, i = 1,2, . . ., 5 делит (ргр2 . . . ps )'о и р«^«« . . . p«j , но не делит а0. Полученное противоречие показывает, что многочлена, удовлетворяющего условию задачи, не существует. 108. Пусть a — корень многочлена Р (х) = апхп + ап-гх"-1 + . . . + ахх + а0. Тогда Y а будет корнем многочлена Q (х) = ап хтп + ап-гх^"-» + . . . + аххт + а0. 109. Число V 2 + V 3 —корень квадратного трехчлена Р (х) = (х- (УТ-VT)) (х - (У~2 +_ V"3")) = = (х— V 2)2 — 3=x2— поэтому Y 2 + Y 3 будет корнем многочлена 4-й степени: (х2 — 1 + 2УТ*) (х2 — 1 —2/Tjc) = (х2 — I)2 — 8х = = ^4— 10*2+ 1. 110. Пусть a — корень трехчлена ахх2 + bxx + cv Второй корень этого трехчлена обозначим через а1# Пусть р — корень квадратного трехчлена а2х2 + b2x + с2. Второй корень этого трехчлена обозначим через р1в Рассмотрим многочлен Р (х) = а\ а* (х - ар) (х - аро (х - ахр) (х - а^). Очевидно, что ар — корень этого многочлена. Запишем многочлен Р(х) в виде 100
Р (х) « р4** + Рзх* + р%х2 + рхх + р0. Используя теорему Виета, вычислим коэффициенты Ръ Ръ Ро' Рз = а\а 2 (— аР — aPi — aiP — aiPi) = — а\ а| хф + ро o?^f—Ь-U—? = а\ aff-^- (aj + а*) +-^- (р^ + р«) + 2Щ = а?а2 х X «Л/ —2а1а2г1с2 Pl = — а\а\ ]1 ахаг аха2 ро= а\а\а*р*\$\ = с\с\. Учитывая равенство pi = a\a\, получаем, что все коэффициенты Р (jc) — целые числа. 111. Пусть a — корень трехчлена ахх2 + Ьгх + съ а Р — корень трехчлена а2х2 + Ь2х + сг% Вторые корни этих многочленов обозначим через oti и рх соответственно. Рассмотрим многочлен Q(x)= a*al(x-(a+$))(x4Pi+Mx-(*+№ (^-(«i+Pi». Очевидно, что a + p — корень этого многочлена и коэффициент при хА равен а\а\. С помощью теоремы Виета вычислим другие коэффициенты Q (х) = а* ар* + q*x* + q2x* + qxx + qQ и убедимся, что это целые числа: 101
= а] а\ (-2 (а + ах) - 2 (р + Pi)) = 2а] а» (А. ? = а2 а2 ((а + р) (ах + рх) + (а + ра) (ах + р) + (а + + Р + «1 + Pi)2) = а\ а\ (2аа1 + 2ррх + (а + а,) (р + рх) + = а^ а\ (^- + ^- + &- а1а 1а2 дг = - а2 о| (а + 04 + Р + рх) ((а + р) (04 + pt) + + (« +Pi) («1 + Р)) = - а] а\ ((а + ах) + (р + fc)) (2aat + + (а + «х) (Р + Pi)) = ~-а\а\ {—Ъ- - ^~)(^ \ а а2 /\ а1 2 2 I q0 = а2 а\ (а + р) (ах + рх) (а + рх) К + р) - а\ а\ х (арх + - + МX X ((оса! + рр + «iP)(aiPi- bib2 а а * = а\ с\ + 2аг — 4а1а2с1с2 -f 1) + (' -hap» Xl((P" CL2C1C2 - - a2bfc2 112. Функция an-\X- aai + PPi) (a + аг) (Р t-pi)2-2pp1)+pp1((a -j- a2 c^ + bbi (a^Cx -\- a -i +а„_2д:-2 + . . . + -1) + аох~п монотонно убывает от оо до О при х, изменяющемся от О до оо. Допустим, что g{b)= 1. Покажем, что Ь — корень /(*). 102
f (b) = bn — an^xfr-1 — . . . — a±b — aQ = bn (1 —an^b^ — —... —aib-in-1) — aob~n) = 0. Если с — не равный Ь положительный корень f(x), то, как нетрудно заметить, g(c) = 1, что невозможно в силу монотонности g(x). 113. Разделим а. на т с остатком: at = q.m + r.t где 0 <; rt < m. Заметим, что хт — 1 = (л; — 1) / (л:). Далее, xat = xqim +г< = /ФЛ"1 — 1) + xri. Но х* = (а:— 1) х X f(x)+1. Следовательно, Xя?* = Д- W f (х) +1, где Д (х) — некоторый многочлен от xt и мы можем записать g(jc) в виде = G(x)f(x) + (лЛ +/• + ... + /«). Таким образом, g(x) делится на f(x) тогда и только тогда, когда на / (а:) делится многочлен h (х) = хГг + хг% + + . . . + хгп. Но, как легко заметить, Так как степень h(x) не превосходит степени f (x), то h(x) делится на f (х) тогда и только тогда, когда h(x) = = cf(x), где с — действительное число. Из этого следует, ЧТО По = Mi = . , , = Лт-1. 114. Предположим, что существуют два таких многочлена / (х) и ф (х) с целыми коэффициентами, степени которых меньше nt что (х — ах) (х — а2) . . . (х — ап) — 1 = / (*)q> (x). Тогда при ^ =■ alt а2, . . ., ап f Ы Ф (fli) = / (я2) Ф W = • • • = / М Ф (ап) = — 1, т. е. f(ai) = —ф(а*) при любом f = 1,2, . . ., п. Многочлен t(x) = / (а:) + Ф (х) имеет м различных корней, хотя его степень ^ п — 1. Поэтому / (л:) = 0 и / (х) = — Ф (х) для всех х. Значит, 103
(x — al){x — a2). . .(x — an)—\ = — Полученное равенство противоречиво, так как при. хп стоят коэффициенты с разными знаками. 115. Предположим противное: sin I ° — рациональное число, т. е. sin 1° = —, где т и п — целые числа. Тогда п cos21° = 1 — sin21° и cos2 Г — sin21° = cos 2° тоже числа рациональные. Аналогично получим, что cos 4° = 2cos22°— — 1, cos 8° = 2cos2 4° — 1, cos 16° = 2cos2 8° — 1 тоже рациональные числа. Далее 3QL = cos 30° = cos (32° — 2°) = cos 32°cos 2°+ + sin 32°sin 2° = (2cos2 16° — 1) cos 2° + 24 cos 16° cos 8° x X cos 4° cos 2° sin 2°. Получили противоречие, так как слева стоит иррациональное число, а справа рациональное. 116. Предположим, что данное число рациональное. Тогда его десятичное разложение, начиная с некоторого места, должно быть периодическим. Далее, очевидно, что период не может состоять из одних нулей. Предположим, что длина периода равна t. Однако данное число содержит в своей записи число КК+1 и поэтому имеет t + 1 нулей подряд. 117. Утверждение будет доказано, если мы покажем (см. задачу 116), что для любого / в записи данного числа встречается более / нулей подряд. Пусть Р (х) имеет вид Р (х) = апхп + ап-ххп-х + . . . + ахх + а0 = = (ап хп~1 + an-ixn-2 + . . . + ах) х + а0. Тогда при х = 10* , где п > / + /, а / — число знаков а0, число Р(х) имеет в своей записи более t нулей подряд. 118. Предположим противное: пусть е= —, где т и q — целые. Умножим обе части равенства 104
на целое число n\q. Тогда в левой части получим целое число п\т, а в правой — сумму целого числа \2-\ (- + — +... Н )n\q и числа q ( 1 l- [- 3! п\) ч ч \п + \ ^ (п + \)(п+2) г + . . • +——— -—■—■ +...]. Мы придем к пропг (л + 1) ...(" +Р) ) воречию, если покажем, что при некотором п второе слагаемое меньше 1. В самом деле, (л + 1) (л + 2) ^ (л + 1)а ' (л + 1) (л + 2) (л + 3)^ (П + 1)3 ' ' '# 1 < 1 Поэтому Если п > <7» то — < 1. л Следовательно, е — иррациональное число. 119. Пусть существует такой квадратный трехчлен ах2 + Ьх +с, где а, 6, с — целые, что аг2 + be + с = 0. Тогда ае + Ь + се"1 = 0, и мы получаем, используя представление е и е~1: 105
п\ .) - -ь. 31 Умножив обе части полученного равенства на N\, где N— некоторое натуральное число, получим: ^N1+ а — с 3! N1 + 2) ' ••• л Г • • • • При iV>|a|+|c| данное равенство противоречиво, так как 2) а 1 + I с\ N + 1 ...+ (N + \)k в(аг,Ь2) Рис. 7 и мы получаем, что целое число равно нецелому. Следовательно, число е не может быть корнем никакого квадратного трехчлена g целыми коэффициентами. 120. Пусть такой треугольник существует (рис. 7). Пусть S— площадь треугольника ЛВС Тогда 1С6
= (a3 — аг) (b2 — b3) — -£-((a3 — a2) (b2 — b3) + ((%— ax) x X (h-bs) + (пъ- т. e. S — рациональное число. С другой стороны, т. е. S— число иррациональное. Полученное противоречие доказывает, что таких треугольников нет. 121. Пусть а = —. Тогда {an} = \-^—\ принимает я { я ) значения вида —, / = 0,1, . . ., q—1. Значит, внутри Я любого отрезка [ 1 ; — нет точек по- L Я V q VJ следовательности an = {ari}. 122. Пусть интервал ] с; d [, содержащийся в ] 0; 1 [, имеет длину а. Тогда среди N точек {an}, n= 1,2, ..., Af {N — любое натуральное число, превосходящее —) а ) обязательно найдутся две, расстояние между которыми менее —. Пусть это точки {а&} и {а/}, />&. Они не совпадают, так как из равенства {ak} = {al} следует, что а имеет вид —, где т и п—некоторые целые числа, П т. е. а рационально. Тогда {(/ — k) а} < а или {(/—k) a} > >1—а. Пусть, например, {(/ — &)а}<а, тогда внутри интервала ]с, d[ оказывается точка {(/—k)ta} при t = И(/Да} J+ 1ш 123. Докажем, что в любом интервале из ]0,1[ содержатся даже члены последовательности an={lgn}, получающиеся при n = 2k , k = 1,2, ... .В самом деле, Ig2fe = = k lg 2, и требуемое следует из рассуждений, проведен- 107
ных в задаче 122, поскольку lg 2 — число иррациональное. 124. Выберем натуральное число &, такое, что сгк < < ~~а . Тогда существуют по крайней мере три нату- 4 ральных числа: Л, А + 1, А + 2, что — 6]я> &L ^"t1 € 1 а, Ъ[, 4+_Lg jfl> ft [. Пусть запись числа А в системе счисления с основанием q имеет вид А = а*.!?*-1 + а*_29*-2 + . . . + «tf + а0. где все щ (i = 0,1, . . ., k— 1) удовлетворяют неравенству 0^а<:^<7— 1. Запишем подряд все целые числа 0,1, . . ., q— 1. Затем к ним припишем всевозможные размещения из 2q чисел {0; 1; . . .; q— 1; 0,1, . . . , q—1} по 2, затем к получившемуся набору припишем всевозможные размещения из 3^ чисел {0,1, . . ., q— 1, 0,1, . . ., q— I, 0,1, . . ., q — 1} по 3 и т. д. Пусть при этом получим {0,1, . . ., 9—1, Pi, 02> Рз> . . •}, где веер, удовлетворяют неравенсту 0^р, ^?— 1. Положим я я* г Ясно, что в разложении а по степеням q имеет место • • • Н г~ Н 717 + • • • Н i, *. 1 Н ггг + • • •» ^ ql+l поэтому {а^-1} имеет вид afe-i ! afe-2 | | a0 | 0 j^ Я Я2 qk qk+{ Получившееся число заключено между —г- и + и Я Як поэтому принадлежит ]a, b[. 108
125. Требуемое утверждение будет следовать из двух несложных фактов: последовательность"^ п (п = 1,2, . . .) неограниченно возрастает; разность между соседними членами последовательности может быть сделана сколь угодно малой за счет выбора достаточно большого п. Первый факт очевиден, а второй следует из неравенства 126. Докажем, что 1\тхп существует. В силу нера- П-+°о венства о среднем геометрическом и среднем арифметическом или к-2 (\ — xn) + xn^2V (l—xn)xn > Следовательно, xn+\^xn, т. е. последовательность xl9 x2t ... неубывающая. Так как по условию она ограничена, то lim xn существует. Обозначим этот предел че- рез х. Устремляя п к бесконечности в обеих частях неравенства (1 — хп) хп+х > —, получаем (1 — х) х > — — х Н =( х ) ^0. Следовательно, х = 4 V 2) 127. Покажем, что HmS,, существует. Действительно, 22 2п 2п^ 2 п п + \ 2i 2а , , 2п' 109
Аналогично Sn+2 = "^ (5"+i + !)• Следовательно, « (п + 1) (п + 3) (5Л+1 + 1) + (« + 2)2 2(п+ 2(л+1)(л + 2) Следовательно, если Sn+\—S^^O, то и Sn+2—Sn+i ^ < 0, так как, очевидно, Sn > 0. Непосредственное вы- 3 5 числение показывает, что «Si = 1, S2 = —, S3 = —, 2 о S4 = —, «S4—S3^0. Индукцией по п заключаем, что о последовательность «S4, S5, ... убывающая. Но Sn>0. Следовательно, lim S п существует. Обозначим этот пре- дел через S. Устремляя п к бесконечности в обеих частях равенства Sn+i = ——^-— (Sn + 1) (см. выше), получаем S = — (S + 1), откуда S = lim Sn = 1. 2 П-»оо 128. Заметим, что {х + п} = {х} для любого целого п и {— х) = 1 — {л:} для любого нецелого х. Из формулы Ньютона (2 + уТ)я = 2« (2 — Следовательно, (2 + ]/~2> = Л + В|/"~2~, (2 — УТ)Л = = А — ВУ^Т для некоторых целых А и В. Тогда {(2+ /"2)"}={Л + ВУТ}= 1— { — A—BVT}= = 1— { А — В/Т}= 1 — {(2 —VT)*}. ПО
Но 0<2 — У 2 < 1, следовательно, lim {(2— уТ)п } = = lim(2 — У~2)п = 0, откуда lim{(2 +У"Т)п) = 1. П-+оо П-+оо 129. Докажем сначала следующее вспомогательное утверждение (лемму). Пусть хъ х2, ...— такая последовательность, что хп >0. Тогда lim хпп = х > 0 в том и только в том случае, ког- да lim п (хп — 1) = In x. П->оо Действительно, если lim xnn = х, то в силу непрерыв- П-*-оо ности функции у = In xy Inх = lim In (х%) = limn In х„. Рас- Л->оо П->оо смотрим последовательность уи у2, . • •, определенную следующим образом: 1, если хп = 1; Уп In* —-3,, если хпф 1. Тогда п\пхп = п (хп — 1) уп для любого п. Следовательно, \imn(xn — 1)уп = \г)х. Но так как lim п \п хп = In x< < со, то liming = 0. Следовательно, Нт;с„ = 1 и 1п х lim г = 1, где m пробегает множество таких /г, что ^л=т^1. Из этого следует, что \\туп= 1. Значит, limnx П-> оо П-> оо X (хп — 1) = In лг. Обратно, пусть lim п (хп — 1) = In x, тогда lim хп = 1 и lim уп = 1. Но п In хп = п (хп — 1) уп , П->оо П->оо откуда lim /г In д;л = In x и Нт^=л:>0. П-+оо П-+оо Теперь переходим к решению задачи. Положим хп = = Рап + gbn. В силу нашей леммы lim п (ап — 1)= In a, Но 111
п (хп — 1) = п (рап + qbn — 1) = рп (ап — \) + qn {Ьп — 1) и в силу леммы In (lim Хп) = lim п (хп — 1) = р In a + q In b = In apbqt следовательно, lim (pan + qbn )n = aP bq. 130. Пусть c> 1. Выберем такое натуральное m, что c<2m. Тогда /(c0>/(0, /(rf)</(2m0. Далее, t< /(gQ ^ /(^mO = /(%) /(220 /(2mp "" ДО "^ /(0 "" f(<) " /(») '"'" /(г1»-1') Каждый из т сомножителей правой части стремится к 1 при tf стремящемся к бесконечности, что следует из определения функции f(x). Значит, отношение ' стремится к 1. Если с<1, выберем такое т, что 2-'п< <с. Тогда f(ct)^f(t), /(tf)>/(2~m/). Из неравенства i^/(0 ^ /(О ДО /(2-[0 /(2-^-!) о ^ f(ct) ^ f(2~mt) опять заключаем, что lim f(2~mt) <-« /(/) = lim П->оо /(О = 1. 131. Вычислим сначала Rn Пусть О—центр всех окружностей,* АВ —сторона 2П+1 -угольника, К — середина [АВ] (рис. 8). Тогда \0А\ = = Rn , | О/( | = /?п+ь Так как ЛОВ= 2я я ТО Rn+l = Кл COS "Г^ГГ . = 2„+1 Отсюда, лучаем: учитывая, что /?х = 1, по112
Преобразуем это выражение. Умножив равенство на sin -^-, получаем: Rn sin —— = cos — cos — ... cos -^- sin -2- = 2n 4 8 2" 2" 1 л я л . я 1 я я = — cos — cos—.. .cos г sin г = — cos — cos —... x 2 4 8 2n~l 2n~x 4 4 8 я . я 1 . л; г» о Xcos—— 51п-^= ... =—rsin—; Rn =± Ясно, что lim-^- = 0. Учитывая, что lim sin* = 1, по- 2 лучаем \imRn = —. 132. Выберем среди чисел аъ a2t ..., ak наибольшее по модулю; пусть это число а1и Тогда из условия следует, что для любого нечетного п В силу выбора ах для любого i = 2, 3, ..., k — < 1. Предположим, что ни одно из чисел— не равно —1. Пусть для i = 2, 3 t9 где / < k, — = 1, а для i = = /+ 1, .... ^ — < 1. Тогда при любом нечетном п должно иметь место равенство 5 Зак 2126 113
Взяв п достаточно большим, можно обеспечить выполне- ние неравенства —) < для каждого ь = \\<h / I Ь — t = t+ 1..., k. Это следует из того, что лучим а{ < 1. Тогда по- что противоречит написанному выше равенству. Следовательно, среди — хотя бы одно число равно —1. Пусть — =—1. Тогда а1 + а2 = 0и k—2 числа aZi а4, ..., ak удовлетворяют условию задачи. Повторяя эти рассуждения, мы при четном k разобьем числа на пары взаимно противоположных чисел, что и требовалось в задаче. Если же к нечетно, то получим, что для одного из наших чисел любая его нечетная степень равна нулю. Но это противоречит тому, что все числа отличны от нуля. 133. На первом этапе мы в ах из k ящиков поместили по k ящиков, на втором этапе в а2 из этих меньших ящиков поместили по k ящиков и т. д. Тогда число т непустых ящиков равно аг -f #2 + • • • + ап - Всего же ящиков k + каг + ka2 + ... + kan = k + km = (m + \)k. 134. Заметим прежде всего, что при выполнении операции над числами, записанными на доске, количество нечетных чисел либо остается неизменным, либо уменьшается на 2. Поэтому, если в конце процесса мы получили нули, то количество нечетных чисел среди 1, 2,..., N было четным. Если N четно, то количество нечет- N ных чисел среди 1, 2, ..., N равно —, если же N не- 114
четно, то оно равно . Таким образом, либо —, либо • должно быть четным числом, откуда следует, что N равно либо 4&, либо 4k— 1, где k—натуральное число. Покажем теперь, что это необходимое условие является также и достаточным. Если Af = 4fe, то мы сначала выполняем операции 2—1, 4—3, ..., 4k— (4k— 1). Получаем 2k единиц; попарно вычитая их друг из друга, получаем нули. Если N = 4k — 1, то выполняем сначала операции 5—4, 7—6, ..., (4k — 1) —(4k —2). Полученные 2 (k — 1) единиц заменяем, как и в предыдущем случае, нулями. Затем после операции 3—1 получаем, кроме нулей, две двойки, вместо которых можем записать нуль. 135. Если 2п£В, то положим А = {/г}. Тогда А+А = = {2п} а В. Если же В не содержит четных чисел, то для А = {2} А + А = {4}, откуда (А + А) П В = 0. 136. Такого множества не существует. Чтобы убедиться в этом, докажем, что если для каждого а £ М и произвольного действительного числа г>0 существует по крайней мере один такой элемент b £ М, что \а — Ь \ = г, то в М найдется три таких элемента х, у, z, что \у — — х\ = \у—г\. Пусть а(М и 6, с—такие элементы М, что \а —Ъ | = 1, \а—с\ = 2. Если а< Ь <с или с<6< <а, то \Ь —с\ = \Ь —а\ = 1. Если 6<а<с или с< < а < 6, то найдем такой элемент d^Mt что \а —d \ = = —. Тогда, если d<a, то \d — 6| = |d — я| = —\ если d>a, то \d — b\ = \d— c\ =—. 137. Каждое число встречается в таблице 101 раз. Из симметрии таблицы следует, что вне диагонали каждое число встречается четное число раз. Поэтому каждое из чисел встречается на диагонали нечетное число раз, 5* 115
следовательно, хотя бы один раз (не нуль раз). Чисел столько же, сколько и клеток на диагонали, поэтому каждое число встречается на диагонали один раз. 138. Пусть ABCD—данный четырехугольник, £, F^ G, Н—середины его сторон, К—середина диагонали BD, MN—отрезок прямой, преходящей через К параллельно второй диагонали (рис. 9). Покажем, что [MN] совпадает с множеством таких течек О четырехугольника ABCD, что площади четырех- угольников EBFO и HOGD равны —, где S — площадь 4 ABCD. Заметим, что SAABo = 2SA£jb0, SABco = ZSabfo. Поэтому площадь четырехугольника АВСО равна удвоенной площади четырехугольника EBFO. То же верно для четырехугольников AOCD и HOGD. Поэтому нам достаточно показать, что [MN] —множество таких точек О четырехугольника ABCD, что площадь АВСО равна S 116
Пусть О € [МЛ?]. Так как {MN) \\ (АС), то 5ДАос = =* Здакс, следовательно, площади четырехугольников АВСО и АВСК равны. С другой стороны, так как К — середина [BD], то SAABk = — Saabd, 5двск = -г-5лвсоэ т. е. площадь АВСК равна —. Пусть теперь O$[MN]. Обозначим через О' точку пересечения (ВО) и (MN)\ из выпуклости четырехугольника ABCD следует, что О' £ [ЛШ] (рис. 10). По только что доказанному площадь АВСО' равна —. Следова- тельно, площадь АВСО не равна —. Проделав те же рассуждения для диагонали АС вместо диагонали BD, можем заключить, что требуемая в задаче точка является точкой пересечения двух прямых, каждая из которых проходит через середину одной из диагоналей параллельно второй. В силу выпуклости четырехугольника, эта точка лежит внутри него. 139. Покажем, что окружности, вписанные в треугольники, образованные сторонами четырехугольника и одной из его диагоналей, касаются тогда и только тогда, когда 117
суммы длин противоположных сторон этого четырехугольника совпадают. Как нетрудно заметить, отсюда следует требуемое утверждение. Пусть в четырехугольнике ABCD проведена диагональ А С; /Ci—точка касания [АС] и окружности, вписанной в ААВС\ К2— точка касания [АС] и окружности, вписанной в AACD; М, N, P, S—остальные точки касания (см. рис. 11 и 12, где отдельно рассмотрены случаи выпуклого и невыпуклого четырехугольников). Используя известное свойство касательных к окружности, получаем: Таким образом, | АВ \ + \ CD | = | ВС \ + \ AD | тогда и только тогда, когда | АКг \ + \ К2С | = | АК2 \ + \ КгС\. Последнее равенство имеет место тогда и только тогда, когда /(х == /(2> т. е. когда окружности, вписанные в треуголь- _в ники, касаются. 140. Из рассмотрения треугольников (рис. 13) DAB и DA'B, BCD и BCD вытекает, что А1 Рис. 13 Отсюда получаем, что ^ABCD Из рассмотрения треугольников А'ВС и А'В'С, DA'C и D'A!C получаем: 11 — Q, — Q 1 — о " ,. = \-(а + Ь). 118
Рис. 14 14L Проведем через В прямую, пересекающую [АС] в точке М так, чтобы ^ ABM s ^ CBD (рис. 14). Углы ВАС и BDC конгруэнтны, как вписанные и опирающиеся на одну дугу. Поэтому треугольники АВМ и BCD подобны, откуда \АВ\ • | СГ> | = | ЛЛ41 • \BD\. Аналогично устанавливается подобие треугольников ВСМ и ABD, откуда следует |ЯС|- \AD\ = \MC\ -\BD\. Складывая эти равенства и учитывая, что | AM | + | МС |= = | АС |, получаем требуемое в задаче. 142. В дальнейшем через X, У, Z, W будем обозначать точки Л, В, С, D, взятые в некотором порядке. Пусть точки Л, В, С и D образуют четырехугольник XYZW (рис. 15). Выразим его площадь через расстояния от точки О до точек X, Г, Z и W. Предположим, что, проходя последовательно по вершинам X, Y, Z, W, мы обхо<- дим контур четырехугольника по часовой стрелке. Введем aXY так: аху= XOY, если луч \0Y) получается из луча [ОХ) поворотом вокруг О по часовой стрелке, и aXY = — XOY, если [OY) получается из [ОХ) поворотом против часовой стрелки. Аналогично определяются ayz, <xzw и awx. Затем, разбив четырехугольник XYZW диагональю на два треугольни- 119
ка, получим $xyzw + w*sinawx)9 где* ■Y(xysinaXY У2 s*n aYZ "Ь |ОХ|, y=\OY\, г =< \OW\. Ясно, что это выражение принимает при aXY = aYZ = Q<zw + 2Шsinaz = |OZ|, w максимальное значение р XY YZ zw wx = 90°. Другими словами, если четырехугольник XYZW максимален по площади, то он выпуклый, его диагонали перпендикулярны и пересекаются в точке О. Предположим, что X = А. Чтобы ответить на вопрос, сформулированный в задаче, нужно выяснить только, какая из трех точек В, С, D должна стоять на месте Z, для того чтобы площадь четырехугольника была максимальной. Докажем, что Z = D. Действительно, если Z = = Д то Sabdc = — (ab + bd + cd + ас). Если Z = В, то Sacbd = — (ас + be + bd + ad). Если Z = С, то Sabcd = = — (ab + be + cd + ad). Имеем SAbdc —SACbd = — X X (ab + cd — be — ad) = — (a — c) (b — d) > 0, Sabdc — — Sabcd = —(bd + ac — be —ad) =—(a—b)(c— d)>0. Таким образом, ответ следующий: через точку О проводим две взаимно перпендикулярные прямые, на одной из них по разные стороны относительно О помещаем точки Д иД па второй — точки В и С, также по разные стороны относительно О (рис. 16). 143. Да. Пусть а — меньшая сторона, b—боль- Рис. 16 12Э
шая, S—площадь прямоугольника. Пусть a<S (если <z^5, то из этого прямоугольника выделим такой прямоугольник, чтобы меньшая сторона была меньше S, и дальше будем рассматривать его). Разобьем а на п равных частей: а = пх\ представим Ъ в виде Ъ = тх + е, где т — целое число и е < х. S = = тпх2 + аг < тпх2 + Sx, откуда . S — Sx S S тп > = — . X2 X2 X В каждый из квадратиков ххх впишем по кругу радиуса —. Сумма радиусов равна — т/г> —( )= 2 Г г 2 2 \х2 х I 2х . Взяв п достаточно большим, добьемся того, чтобы S S было больше 1978. Затем, i e колько уменьшив 2х 2 J радиусы, получим их сумму, равную 1978. 144. Оси координат делят плоскость на четыре части. По условию каждый из данных четырехугольников целиком лежит в одной из них. Следовательно, найдется бесконечное множество Mt состоящее из прямоугольников, которые лежат в одной из четырех частей плоскости, например в первой четверти. Если каждый прямоугольник из М содержится в некотором другом прямоугольнике из М, то не составляет труда построить требуемую последовательность. Предположим, что существует прямоугольник Р б М, не содержащийся ни в каком другом прямоугольнике из М. Обозначим через Мх множество прямоугольников из М, которые ниже Р, через Му — множество прямоугольников из М, которые уже Р. Тогда Мх U Му содержит все прямоугольники из М, отличные от Р. Множество Мх (]МУ конечно, так как состоит из тех прямоугольников из М, которые содержатся в Р. Таким образом, либо Мх, либо М бесконечно. Пусть бесконечно множество Мх. Тогда существует такое нату- 121
ральное N9 что множество прямоугольников из Мх, высота коюрых равна N9 бесконечно. Занумеровав прямоугольники этого множества в порядке возрастания ширины, получим требуемую последовательность. 145. Так как точки С, В, А' и D' лежат на окружности с центром О, то CDB = 2CDB, BOA' = 2BDA', откуда С О А' = СШ + BOA' = 2 {С'Ъв + BDA')=2(90°— — ЛВЬ + 90° —DBC) = 2 [180° —(ABD + DBQ] = 2B/1D. Так как вокруг каждого из четырехугольников ACB'D и DB'A'C можно описать окружность, то C'B'A=C'DA= = 90° — ВЛД AfJBfC = А'Ъс = 90° —Af/CD = 90° -ВлЪ, откуда СКА' = 180° — С^Л —АЧЗ'С = 180° — (90° — — BAD) —(90°—BAD) = 2BAD. Таким образом, ^С'ОА'= ^^С'В'А'. Отсюда следует, что точки А', В\ С и О лежат на одной окружности. Рис. 17 Рис. 18 146. Для удобства обозначим a = \BC\f Ь = \АС\У с=^ = \АВ\. Рассмотрим четырехугольник AMLO (рис. 18). Так как АМО = ALO = 90°, то точки М и L лежат на окружности с диаметром [АО]. Применяя к четырехуголь- 122
нику AMLO теорему Птоломея (см. задачу 141), получаем: | AM |. | LO | + | ML | • \АО | = | AL\ • {MO |. Подставив в это равенство | AM | = —, | AL \ = —, | ML | = —, | МО| = dc, | L01 = — dB, \AO\=Rt получаем после преобразования bdc cdB aR ~1Г + НГ~ = ~Т"- Аналогично из четырехугольника OLKC получаем: bdA , д^в __ _£^_ 2 + 2 2~# Четырехугольник ОМВК также может быть вписан в окружность, ибо М = /С = 90°. Тогда по теореме Птоломея получим: ~т~ 2 Далее, так как S&abc = Saabo ~j~ Saobc—Saaco* to Saabc = -|-1 OM | + y 10/CI T' °L '* C другой CTopo" ны, SAAbc = (g +c)rt Отсюда следует, что adA bdB cdc (a + b + c) r Складывая четыре полученных равенства, имеем после а-\- b -\- с сокращения на —! —: 123
147. Пусть ABC—вписанный в окружность треугольник, Р—точка на окружности, S, м, N — основания перпендикуляров из Р на стороны треугольника (рис. 19). Так как AMP = ASP = 90°, около SAMP можно то описать /\ /\ окружность. Тогда Л MS = APS, как опирающиеся на одну дугу. Аналогично NMC = NPC. Далее, так как четырехугольник РАВС— вписанный, то РАЗ + + РСВ= 180°. Тогда JVPC = = 90° — РСВ = 90° — (180° — _ РАВ) = РЛВ —90° = APS. Рис. 19 Следовательно, == Л^УИС, откуда и следует, что точки S, Л1 и Af лежат на одной прямой. 148. Поскольку АС В + FCD = Р?С + + FCD = 180°, то ^PFC^^ACB (рис.20). Аналогично -^ BKQ ^ ^-^СВЛ. Далее, так как [FP]^[CD\^ICB\, [FC] ^ [АС] (мы использовали то, что AGFC и DEBC — квадраты, a FPDC — параллело- Рис. 20 124
грамм), то Л FPC ^ Л KBQ. Аналогично Л BKQ ^ ^ Д ABC. Значит, ^FCP ^ ^KBQ з* ^СЛВ, откуда следует, что ^ACP^^ABQ. Но [РС]о*[АВ], [BQ]^> &[АС] (из конгруэнтности треугольников FBC> ACB и BQ/Q. Следовательно, /sAPC ^ AABQ. В частности, мы доказали, что [AP]^[AQ]t т. е. что APAQ—равнобедренный. Далее, так как ^APC^L^BAQ, то + АСР + РЛС = b'aQ + Р^С + АСР = 180°. АСР = 90° + FC*P = 90° + СЛВ. Следовательно, PAQ = BAQ + РАС + CAB = 90°, т. е. &PAQ —прямоугольный. 149. Пусть \АВ\ = с, \АС\ = Ь, \ ВС \ = а. Из условия следует, что а<6<с (рис. 21). Пусть М9 Lt N—основания перпендикулярэв, опущенных из точки / на прямые (ЛС), (ВС) и (АВ) соответственно. Прежде /\ /\ всего заметим, что 1ВО<1СВО<С /\ < 90°. Далее, если IOB = 90°, то, так как О—середина отрезка [ВА], N совпадает с О и AlAO^ ^Л IBO, откуда ^IAB ^ ЛВА и ^-САВ^^-СВА, но по условию CAB <СВЛ. Следовательно, /ОВ=£ =^90°. Таким образом, если ДЮВ — /\ прямоугольный, то ВЮ = 90°. Рис. 21 Далее воспользуемся известными формулами: \AM\ = \AN\ = + с — а а + с — Ь 125
[в частности, так как ——*> —, то точка N лежит между точками О и В). Следовательно, 2 2 2 = |/ЛП2 + | Щ 2 = (" 2аг + 26г + сг — 2ас — 26с 4 |B/|2=|BZ,|2 + |L/|2 = 2а2 + 262 + 2с2 — 46с И если О/В = 90°, то по теореме Пифагора или 2а2 + 262 + с2 —2ас—2Ьо + 2а2 + 2Ь2 + 2с2 — 4Ьс=с2. Приведя подобные члены и сократив на два, получим: 2а2 + 262 + с2 —ас—ЗЬс = 0. Но а2 + Ь2 = с2, откуда Зс2 = ас + 36с, Зс = а + 36. Таким образом, имеем: 13с = а + 36, \с2 = а2 + б2. 126
a b Полагая, что х = —, у = —, получим систему: с с + 3у = 3, 4 Эта система имеет два решения: 1) х = —1, У = — Первое решение геометрического смысла не имеет. Следовательно, — = —, — = —, т. е. с 5 с 5 \ВС\: 150. Пусть О— с центр вписанной окружности, М —точка пересечения медиан, Q и Р—точки пересечения прямой (ОМ) со сторонами АС и ВС (рис. 22). Обозначим через г радиус вписанной в Л ABC окружности. Тогда, очевидно, Л CQO se ^АСОР, \CQ\ = \CP\ и S&CQO == 5 Пусть р—расстояние от точки М до прямой (ВС), д— расстояние от точки М до прямой (АС). Тогда, поскольку Sacqp == «Sacqo ~Ь «^асро = *Sacqm ~h « и 127
^ = Tp'|CP| = Tpx XICQI, то получаем, что p + q = 2r. Пусть ha и hb — высоты, опущенные из вершин А и В соответственно. Так как М—точка пересечения медиан, то нетрудно заметить, что р = — ha, q = — Л6. Значит, 2г = д , но А 3 = ^, откуда . (а + Ь + с) г . (а+Ь + с)г К = . hb = . Следовательно, ( ) К = . hb откуда или 3 -L + -L 3 а + Ь а ^ b 151. Пусть дан ААВС. В нем \АВ\ = п — 1, |ВС|== = я, | ЛС | = п + 1, где я —целое число, п > 4 (рис. 23). Применив формулу Герона, получим: с <, /21/З/г2— 12 о = оьавс = Так как по условию S—целое число, то Y3n2 — 12 — рациональное. Нетрудно показать, что если У"/л (где т— целое)—рациональное, то rfm —целое число. Действительно, пусть Vm —• —, где р и q—целые числа и не я а имеют общих делителей. Тогда -£—= /п— целое, и, по-
скольку р2 и q2 не имеют общих делителей, то q = 1, откуда У~гп = р — целое число. Таким образом, У^Зп2—12^ целое число. Далее, если п нечетно, то нечетны и числа 3/г2 — 12, V "3/г2 — 12 и — 12 . Но тогда S= ТГ12 не мо- жет быть целым. Следовательно, п = 2/п, где m — целое число и S = т Vim2 — 3, откуда УЗт2—3 —рациональное, а значит, и целое число. Рассмотрим высоту AD, опущенную на сторону длины п: \AD\ = 2S ВС\ = Y3m2— 3. —3—целое Так как мы уже показали, что число, то \AD\—целое число. Далее, |СО|2 = |ЛС|2 — \AD\2 = (2m+ I)2 — (Зт2 — 3) =(т+2)2, |BD|2 = | АВ\2 —| AD\2 = (2т — I)2 — (Зт2 — 3) = (т—2)2 и, так как из условия следует, что т > 2, то | CD | = = m + 2, | BD | = m — 2 — целые числа. Кроме того, | CD | — | BD | = (m + 2) — (m — 2) = 4, что и требовалось доказать. 152. Воспользуемся известными формулами: 2с2 — а2 be ((Ь + с)*— а«) (ft + c)« 129
(Если вам не встречались эти формулы, попробуйте их вывести сами.) Тогда та __ (6 + с)2 262 - \\ "" 46с (b + cf — i —а2 Но 262 + 2с2 — (6 + с)2 = (Ь —с)2>0, откуда 2ft2 + 2с2— — a2^(ft + c)2—а2>0. (Мы использовали также то, что Ъ + с > а.) Следовательно, 2Ь2 + 2с2 — а2 ^ - ■ с)2 — а2 откуда Из рассуждений видно, что равенство достигается лишь в случае, когда b = с. 153. Обозначим через М заданное конечное множество точек. Из конечности М следует, что М содержится в некотором квадрате /С. Среди полуплоскостей, определяемых прямыми (АВ), где {А, £}с:УИ, выберем все те, которые содержат М. Пусть это полуплоскости Ьъ L2,..., Ln . Тогда К П ^i П L2 П ... П Ln является выпуклым многоугольником, содержащим М. Это нетрудно показать с помощью индукции по п, пользуясь тем фактом, что пересечение выпуклого многоугольника и полуплоскости является выпуклым множеством точек. Покажем, что любая вершина S построенного Рис. 24 многоугольника принадлежит М. Пусть Р, Q— 130
соседние с 3 вершины многоугольника (рис. 24). Из построения вытекает, что на сторонах [PS] и [SQ] находятся точки изМ. Пусть А £ [PS] и Bf [SQ] —ближайшие к 5 из точек М на этих сторонах. Если внутри AASB находятся точки М, то выберем из них такую точку С, что угол SBC минимален. Пусть R —точка пересечения (ВС) и (PS), тогда из условия выбора С внутри ARSB нет точек М. Следовательно, все точки М находятся в полуплоскости, определенной прямой (ВС), которая не содержит 5; при этом {В, С}аМ. Обозначим эту полуплоскость L. Если же в /\ASB нет точек из М, то одна из полуплоскостей L, определяемых (АВ), содержит М. Таким образом, мы построили полуплоскость L, содержащую М и отличную от Llf L2, ..., Ln, что противоречит построению. 154. Рассмотрим сначала три точки Л, В, С данного множества. По условию все стороны треугольника ABC различны. Пусть для определенности | АС | < | СВ \ < <| АВ\. Тогда из точки А соединяющий отрезок ведется в С, из точки С — в В, а из точки В—в С. Таким образом, для этих трех точек провели два отрезка [АС] и [ВС]г причем, как это следует из неравенств для сторон треугольника, угол между ними больше 60°. Рассуждать будем от противного. Предположим, что в некоторой точке сходится по крайней мере шесть соединяющих отрезков АВЪ АВ2, ..., ЛВв. Из сделанного вы- /\ ше наблюдения следует, что каждый из углов В^Вз, /\ /\ В2АВ3} ..., BQABl больше 60°. В таком случае сумма этих углов больше 360°. Пришли к противоречию, которое опровергает сделанное предположение. 155. Проведем 495 попарно параллельных прямых /ь 4» ..., /495 так> чтобы все данные точки были расположены внутри 494 полос, границы которых—прямые /ь 4» ..., /495> причем внутри полосы, ограниченной /х и /2, Должно находиться пять из данных точек, внутри полос 131
с границами /2 и /3, /3 и /4, ..., /494 и /495—по четыре точки (рис. 25). Покажем теперь, что из пяти точек, никакие три из которых не лежат на одной прямой, можно выбрать четыре, лежащие в вершинах выпуклого четырехугольника. Пусть это точки Л, В, С, D. Используя рассуждения из решения задачи 153, построим выпуклый ^-угольник, где k = 3, 4 или 5, с вершинами в некоторых из этих точек, содержащий внутри себя остальные. Если k = 4 или k = 5, то любые четыре вершины этого ^-угольника образуют выпуклый четырехугольник. Таким образом, по крайней мере один четырехугольник мы можем построить. Пусть k = 3; для определенности предположим, что точки D, Е лежат внутри треугольника ABC. По условию (DE) не проходит через вершину А Л ВС, поэтому (DE) пересекает две стороны треугольника. Если это стороны А В и АС у то точки В, С, D, Е лежат в вершинах выпуклого четырехугольника. Используя доказанное утверждение, можем выбрать выпуклый четырехугольник Ai^2BiB2 с вершинами в четырех из пяти точек, лежащих в полосе между 1г и /2. 132
Точка пересечения ИхВ^ и [Л2В2] лежит, очевидно, внутри полосы между 1Х и /2. Рассматривая оставшуюся неиспользованной точку из полосы между 1г и /2 вместе с четырьмя точками из полосы между /2 и /3, Найдем выпуклый четырехугольник Л3Л4В3В4. Точка пересечения [Л3В3] и [Л4В4] лежит внутри полосы между /2 и /3. Продолжая этот процесс, рассматриваем неиспользованную точку из полосы между 1Х и 1п вместе с четырьмя точками из полосы между 1п и /п+ь находим выпуклый четырехугольник А2п-\А2пВ2п-\В2пу а точка пересечения [А2п-\В2п-\] и [A2nB2n] лежит в полосе между 1п и 1п+{. Поступая таким образом, мы так разобьем 1976 из данных точек на пары, что в каждой из 494 полос будет находиться хотя бы одна точка пересечения отрезков, попарно соединяющих точки. Следовательно, точек пересечения будет не менее чем 494. 156. Возьмем п—2 попарно не подобных прямоугольных треугольников А^Сх, A2B2C2j ..., An-2Bn-2Cn-2, длины сторон которых выражаются целыми числами. Это можно сделать по ранее доказанному (см. задачу 67). Пусть h—наименьшее общее кратное длин катетов АХВ , ^2#2» ..., Л„_2#я-2. Некоторый отрезок [А'В'] длины h 133
возьмем в качестве общего катета треугольников ArBrC'v А'В'С'у ..., A'B'C'n_v подобных треугольникам А^В^С^ А2В2С2у ..., Ап-2Вп-2Сп-2. Ясно, что все расстояния между любыми из п точек А', В', CJ, C'v .. ., С'п_2 выражаются целыми числами (рис. 26). 157. Зафиксируем некоторое число а и построим круги радиусов а с центрами в заданных п точках. Каждую пару пересекающихся кругов можно заменить одним кругом, который их содержит и имеет радиус, не превосходящий сумму радиусов двух исходных (рис. 27). Повторяя эту операцию, придем к совокупности т^п попарно не пересекающихся кругов, содержащих данные точки вместе с первоначальными кругами радиуса а, сумма диаметров некоторых из них не превосходит 2па. Уменьшим теперь радиус каждого из т построенных кру- Рис. 27 гов на некоторое число Ъ < а. Полученные круги по-прежнему покрывают данные точки, сумма их диаметров не больше 2па —2тЬ <; 2па — 26, а расстояния между каждыми двумя не меньше 2Ь. Осталось только подобрать числа а и Ь так, чтобы 2па —2Ь < п и 2Ъ > 1. Этим неравенствам удовлетворяют, например, такие: 1.1, 1,1 158. Доказывать будем индукцией по п (по числу городов). Если п = 2, то утверждение очевидно. Предположим, что мы доказали требуемое в задаче для числа городов, не превосходящего п. Пусть у нас п + 1 город и А — 134
один из них. Обозначим через Д+ множество городов, в которые можно проехать из Л без пересадок, а через Д~— множество городов, из которых можно проехать в Л без пересадок. Если множество Д+ пусто, то Л—искомый город. Если в Д+ только один город, то он является искомым. Пусть число городов из Д+ не меньше двух; так как это число не превосходит пу то по предположению индукции найдется город В из Д+, в который из любого другого города из Д+ можно проехать, сделав не более одной пересадки. Покажем, что В—искомый город, т. е. что из произвольного города С можно проехать в В, сделав не более одной пересадки. Если С £ Д , то это вытекает из выбора В. Если С = Л, то мы можем проехать в В без пересадки. Если С £ Д~~> то в В можно проехать с пересадкой в А, 159. Рассмотрим на плоскости окружность радиуса Y 3 с центром в О. Пусть Р — произвольная точка окружности, ОАРВ—ромб с диагональю ОР и сторонами, равными 1. Предположим, что любые две точки, расстояние между которыми равно 1, окрашены в разные цвета. Пусть точка О скрашена в цвет I, тогда одна из точек Л, В окрашена в цвет II, вторая—в цвет III (ибо треугольник ОАВ правильный со стороной 1). Отсюда следует, что точка Р, которая находится на расстоянии 1 от Л и Ву окрашена в цвет I. Таким образом, все точки окружности окрашены в один и тот же цвет. Нетрудно найти две точки на ней, расстояние между которыми равно 1, что противоречит предположению о том, что такой пары точек не существует. 160. Пусть п—произвольное число, большее 4. Тогда из п точек Съп способами можно выбрать пять точек, а среди этих пяти точек не менее одной четверки точек лежат в вершинах выпуклого четырехугольника (последнее утверждение доказано в решении задачи 155). Однако каждая четверка точек, лежащих в вершинах выпук- 135
лого четырехугольника, будет при этом сосчитана п—4 раза, т. е. столько раз, сколькими способами эти четыре точки можно дополнить до пяти точек. Следовательно, число четверок точек, лежащих в вершинах выпуклого С5 четырехугольника, не меньше - * , что и требовалось доказать. 161. Среди треугольников с вершинами в данных точках выберем ДАВС наибольшей площади. Проведя через вершины треугольника ABC прямые, параллельные противоположным сторонам, получим треугольник Л'В'С, площадь которого равна 45давс < 4. Если мы покажем, что все точки заданного множества лежат внутри или на граьице треугольника А'В'С\ то, увеличив треугольник А'В'С, получим искомый треугольник площади 4. Стороны {А'В'), (В'С), {А'С) разбивают множества точек плоскости, лежащих вне /\А'В'С\ на шесть областей (рис. 28). Предположим, jb что некоторая точка X задан- 11 с \ в л'/& ного множества лежит в области 1а. Тогда 8ьхвс> >S\abc, что противоречит выбору треугольника ABC. То же будет, если точка X множества лежит в области Па. Аналогично доказываем, что ни одна из точек задан- Рис. 28 ного множества не может находиться в областях Ь, Ив, 1с, Не. Таким образом, все точки множества принадлежат треугольнику ArBrC\ 162, Пусть в данном выпуклом многоугольнике М расположено п непересекающихся кругов 5Х, 52, ..., Sn с центрами 0ъ 02, .. ., On и радиусами гъ г2, ..., гп. Каждой точке X плоскости поставим в соответствие п чисел 136
ht (X) = | XOi ]2 — r\, /=1,2,..., п. Обозначим через D± множество точек X £ My таких, что для любого i = 2, 3,..-Л, п h1(X)^.hi (X). Аналогично определяются множества D2, D9, ..., Dn. Ясно, что каждая точка X $ М принадлежит по крайней мере одному из множеств Ьь D2,..», Dn ♦ Покажем, что Dx, Z)2, ..., Dn —выпуклые многоугольники, на которые разбивается М, и что 5^ си Dit /=1, 2, ..., п. Обозначим через Ht множество таких точек X плоскости, что hx (X) <; ht (X), где i — 2, 3, ..., п. Покажем, что Нг—полуплоскость. Пусть i = 2. Найдем такую точку Р £ [ОХО^ что IPOxl2 — rf = | РО212 — г\ (рис. 29). Она существует / . о,О212 — (г? — г?) \ и единственна ( | РОХI = oi ^ ^ . Проведем 210,0,1 через точку Р прямую /, перпендикулярную (ОхО2). Пусть X / | ХО |2 \ | РО |2 | РХ |2 » РО |2 р у ру X € /, тогда | ХОг |2 -г\ = дуру (х2) у + | РХ |2 -г» = | РОЯ |2+ Рис. 23 + I PX |2 — r\ = | X02 \* —r\. Прямая / делит плоскость на две полуплоскости. Пусть точка Y лежит в той же полуплоскости, что и 0ь тогда \Y0x\2— r^lFiOxl2— r\. И так как Кь Y2 принадле- 137
жат /, то можем продолжить; |V\Oi|2—г^ = |KiO2|2—г < I У*0212 - т\ < | Y0212 -1% т. е. hx (Y) < h2 (Y). Анало- гично, если Y принадлежит другой полуплоскости, то hi(Y)^ h2(Y). Таким образом, множество Н2 есть не что иное, как полуплоскость с границей /, содержащая 0х. Построим теперь ряд многоугольников: М2—пересечение М с Н2\ М3— пересечение М2 с H3i ...; Мп — пересечение Мп-\ с Нп. Так как, очевидно, пересечение выпуклого многоугольника с полуплоскостью является выпуклым многоугольником, то все многоугольники М2у М3, ..., Мп — выпуклые. Сравнивая определения Ht и Du легко заметить, что D± = Мп, т. е. Dx—выпуклый многоугольник. Аналогично убеждаемся в том, что D2t D3,..., Dn —выпуклые многоугольники. Точки, принадлежащие двум (и более) многоугольникам Diy лежат на границах построенных выше полуплоскостей и, следовательно, на границах многоугольников. Таким образом, М разбивается на выпуклые многоугольники Dl9 D2, ..., Dn. Для завершения доказательства того, что Db D2, ..., Dn — искомые многоугольники, достаточно показать, что для каждого /= 1, 2, ..., п круг S, содержится в многоугольнике Dt. Покажем, например, что Sx с Dlt В самом деле, если X £ Sb то fti(X)<10, и, поскольку круги не пересекаются, то hi(X)'^>0 для всех остальных i = 2, 3, ..., /г, т. е. X € Dle 163. Заметим прежде всего, что каждый из треугольников, на которые разбит я-угольник, является вписанным в ту же окружность, что и заданный /г-угольник. Пусть О—центр этой окружности, R—ее радиус. Через ?ъ ''г» • •.» гп-2 обозначим радиусы окружностей, вписанных в треугольники разбиения (то, что этих треугольников в точности п—2, нетрудно проверить подсчетом суммы их углов). Для каждого из этих треугольников 138
запишем равенство, доказанное в задаче 146. В каждом из этих равенств участвуют расстояния от О до сторон или диагоналей /г-угольникэ, взятые со Знаком плюс или минус. При этом каждое из расстояний до сторон встречается только в одном равенстве и со знаком плюс. Так как каждая из проведенных диагоналей является стороной двух треугольников, лежащих по разные стороны от нее, то расстояние от О до этой диагонали встретится в двух равенствах и с разными знаками. Поэтому, сложив все равенства, получим: (к + d2 + ... + dn = (п -2) R + г± + г2 + ... + г/г_2, где dlf d2, • • •» dn—расстояния от О до сторон я-уголь- ника. Выразив отсюда сумму радиусов вписанных окружностей, увидим, что она не зависит от того, какие из диагоналей были проведены. 164. Докажем, что этого сделать нельзя. Предположим противное: пусть для некоторых нумераций сторон и отрезков Sx = S2 = ... = S1Ooo = *S> где Sx—сумма номеров сторон /\OAiA2, S2—сторон ДОЛ2Л3, ..., 51Ооо— сторон ДОЛ1()о<И1. Тогда 10005 = = Si +S2 + ... + S1000 = 3(1 + 2 + ... + 1000) = -, откуда S= нецелое число, что 2 ' J 2 противоречит определению 5. 165. Возьмем / попарно не пересекающихся кругов /Сь Къ • • •» Kt диаметра 1 с центрами в М. Каждый из них заменим на концентрический круг К[ радиуса 1. Если мы покажем, что круги K'v К'2, ..., К\ покрывают М, то получим s^/. Предположим, что точка х£М не содержится ни в одном из кругов К\> i= 1, 2, ...,/. Тогда, поскольку х удалена от центров кругов Kl9 К2, . • •, Kt больше, чем на 1, круг Ко диаметра 1 с центром в х не 139
пересекается ни с одним из кругов Kv i = 1, 2, ..., /. Следовательно, можно построить t + 1 попарно не пересекающихся кругов /Со, /Сь ..., Kt диаметра 1 с центрами в М, что противоречит определению /. 166. а) Рассмотрим три последовательные стороны АВ, ВС, CD вписанного /г-угольника (рис. 30). Углы ВАС, ВСАУ CBD и CDB—вписанные и опираются на конгруэнтные хорды. Поэтому ^ВАС ^ ^:ВСА ^ ^CBD ^ , откуда следует, что ААВС^ ABCD. Но тогда , что и требовалось доказать, б) Воспользуемся теми же обозначениями, что и выше (рис. 30). Углы ABC и BCD конгруэнтны по условию, углы ВАС и BDC конгруэнтны, кг к вписанные и опирающиеся на одну хорду. Тогда ААВС = ADCB, откуда [AB]^[CD]. Таким образом, стороны /г-угольника конгруэнтны через одну. Если п нечетно, то этого достаточно, чтобы утверждать конгруэнтность всех сторон /г-угольника. В то же время для любого четного п существует неправильный вписанный /г-угольник с конгруэнтными углами. Вершины этого /г-угольника строим следующим образом. Множество вершин {Аъ Л2,..., Ak} правильного вписанного fe-угольника повернем вокруг центра окружности на угол, не кратный Риг 4ft Л ™L> ou —; получим множество точек k {В1у В2, .. ., Bk}. Тогда точки Аи Ви Л2, В2, ..., Лл, Bk дают вершины искомого 2/г-угольника (проверьте!) На рисунке 31 показан случай для k = 5. 167. Положим АЕС = а (рис. 32). Тогда, поскольку ^.АСЕ опирается на диаметр, то САЕ = 90°—а. Далее, 140
СВА= 180°—а, откуда CDE*=* = 180° — САЕ = 90° + а. Применив теорему коеинусов к /SABC, получим: |ЛС|2 = fa a2+b2—2ab cos (180°—а)= = а2 + Ь2 + 2ab cos а. Применив теорему косинусов к &CDE, получим:. \CE\2 = c2 + d2—2cd х X cos (90° + а) = с2 + d2 + + 2cd sin а. Но | АС |2 + | СЕ |2 = | АЕ |2 = 4, | ЛС | = | АЕ | sin а = = 2 sin а > | ВС | = Ь, | С£ | = 2 cos а > с. Следовательно, 4 = | АС |2 + | СЕ |2 = а2 + Ь2 + с2 + d2 + ab (2 cos а) + + cd (2 sin а) > а2 + Ь2 + с2 + d2 + abc + bed, что и требовалось доказать. 168. Очевидно, достаточно рассмотреть три оси симметрии. Прежде всего заметим, что никакие две из них не параллельны. В самом деле, композиция двух симметрии с параллельными осями представляет собой параллельный перенос, а никакой многоугольник не переходит в себя при параллельном переносе. Пусть О—точка пересечения двух осей симметрии. Композиция этих симметрии является поворо- Рис. 32 141
том вокруг О, который переводит многоугольник в себя. Так как многоугольник не может сохраняться при поворотах вокруг двух различных точек Ох и О2, то отсюда следует, что все Точки попарного пересечения осей симметрии совпадают, что и требовалось доказать. 169. Проведем диаметр АР окружности Sx (рис. 33). Так как ^jAB окружности S2 делит площадь круга пополам, то она не может находиться в одном полукруге. Следовательно, у^АВ и диаметр АР пересекаются во внутренней точке М круга, ограниченного Sx. Ясно, что длина у^АМ больше |ЛМ|. Если провести окружность Рис. 33 Рис. 34 с центром в М радиуса | МР |, то становится очевидным, что длина ^>МВ больше |AfP|. Следовательно, длина к^АВ больше | AM \ + \ МР | = | АР |, что и требовалось доказать. 170. а) Прежде всего заметим, что все грани такого многогранника должны быть треугольниками. Действительно, если Аъ А2у ..., Ak—грани многогранника, то наш многогранник имеет по крайней мере 2k различных ребра (рис. 34), откуда 2&<!7 и &<!3. Пусть п—число граней такого многогранника. Тогда, поскольку каждая грань имеет три ребра и каждое ребро 142
принадлежит ровно двум граням, то 3/г = 2 • 7 = 14. Получим противоречие, так как 14 не делится на 3. Следовательно, не существует многоугольника с семью ребрами. б) Заметим, что при k^3 ^-угольная пирамида имеет 2k ребер, т. е. для любого четного п ^ 6 существует выпуклый многогранник, имеющий п ребер. Но «отрезав» от ^-угольной пирамиды «уголок», как показано на рисунке 35, мы, очевидно, получим многогранник, ис* имеющий 2k + 3 ребра. Следовательно, для любого нечетного п^2-3 + 3 =-- 9 существует выпуклый многогранник, имеющий ровно п ребер. 171. Пусть п—число восьмиклассников, т — число очков, набранных каждым из них. Всего очков в турнире набрано тп + 8. С другой стороны, п + 2 участника сыграют ("+ )(/г+ ) napTHg и наберут столько же оч- ков. Откуда тп + 8 = <" 2тп + 16 = п2 + Ъп + 2, п2 + Зп —2тп = 14, п(п + 3—2т)= 14. Так как п и 2т целые, то п — или 1, или 2, или 7, или 14. Если /г = 1 или п = 2, то два семиклассника не смогут набрать 8 очков. Итак, п = 7 или /г= 14. 172. Пусть R—требуемое в задаче число жителей, £^а, Dв, Ос — число жытеяей городов А, В и С, UAb — число жителей Л, имеющих знакомых в В, Vас, Ubc определяются аналогично, Uabc—число жителей Л, имеющих знакомых и в В, и в С. Тогда R = Uа + Ub + + Uc — 2UAB — 2UAc — 2UBc + 3UABc >UA— 2UBc + + UB— 2UAb + Uc — 2UAc. Из условия: 1) UA = 6000, 143
2) £/вс<2000, 3) С/в— 2UAB>0, Uc — Wас>0, откуда R>UA— 2UBc > 6000 — 2 • 2000 = 2000 > 1978. 173. Выпишем всевозможные разложения 36 на три сомножителя: 36= 1 • 1 • 36= 1 -2. 18= ЬЗ- 12= 1 . 4-9= 1.6-6= = 2.2.9 = 2-3.6 = 3.3.4. Суммы сомножителей этих росьми разложений равны соответственно 38, 21, 16, 14, 13, 13, 11, 10. Второй, зная число окон дома, а значит, и сумму возрастов (2х X 2 • 9 и 1 • 6 • 6), не мог установить возраст сыновей, так как сумма возрастов при этом одинакова. Но вариант 1-6-6 отпадает, так как в этом случае в семье не было бы старшего сына. Остается 2, 2, 9. Василий Иванович Берник, Игорь Константинович Жук, Олег Владимирович Мельников Сборник олимпиадных задач по математике Редактор В. В Амбражевич. Обложка художника В П. Кореш- к о в а. Художественный редактор Н. Л. Ш а в ш у к о в а. Технический редактор В. Н. Ж У к. Корректоры Cf С. А н и с к е в и ч. М. Г. Виноградова. ИБ № 768 Сдано в набор 16.04.79. Подписано в печать 21.11.79. Формат 70X108V32. Бум. тип. № 2. Гарнитура литературная. Высокая печать. Усл. печ. л. 6.3. Уч.- изд. л. 5,17. Тираж 50 000 экз. Заказ 2126. Цена 20 к. Издательство «Народная асвета» Госкомиздата БССР. 220600 Минск, Парковая магистраль, 11. Полшрафкомбинат им. Я- Коласа Госкомиздата БССР. 220827 Минск, Красная, 23.